You are on page 1of 49

CRIMINAL LAW

I. GENERAL
A. Definitions
1. Anything of value: movable or immovable  should be given broadest possible construction
2. Felony: crime punishable by sentence of death or hard labor
3. Person: human being from moment of fertilization, juridical person  BUT crimes like homicide
that talk about “human beings” mean a person that has been born alive
4. Crime of violence: crime that has as an element the use, attempted use, or threatened use of
physical force against the person or property of another and which involves the possession of a
dangerous weapon or which by their nature involve substantial risk that physical force will be used.
B. Sentencing
1. Felony: punishable by death or hard labor
2. Misdemeanor: anything else
a. Misdemeanor where there is RTC: punishable by imprisonment of 6 months or a fine of
$1000.
b. Misdemeanor with no RTC: punishable by less than above.
3. Relative felony: either punishable by hard labor or not, at trial judge’s discretion
C. Mens rea
1. Specific intent: D actively desires the prescribed criminal consequences to follow his act or
failure to act
2. General intent: D, in the ordinary course of human experience, must have known that the
prescribed criminal consequences were reasonably certain to result from his act or failure to act.
3. Negligence: D’s conduct amounts to a gross deviation below the standard of care required of
a reasonably careful person under the same circumstances
4. Recklessness: not specifically discussed in the code
a. If D was aware of the risk: general intent
b. If D not aware: negligence
5. Knowingly: some statutes will make this an element
6. Intentional: some statutes will require that the defendant intended to do the physical act that is the
object of the statute
D. Parties
1. Principal:
a. Definition: a person who
i. Directly commits the act constituting the offense;
ii. Aids and abets another in committing it; or
iii. Directly or indirectly counsels or procures another to commit it.
b. Presence or absence at scene is irrelevant
c. Acts of one principal are attributed to other principals, but must show each one had
the appropriate mens rea.
d. Exception: can’t convict buyer of drugs on conspiracy to distribute drugs.
2. Accessory after the fact:
a. Definition: Person who after another has committed a felony,
i. Harbors,
ii. Conceals, or
iii. Aids the felon;
iv. Knowing or having reasonable grounds to believe that the person committed
the felony.
b. Need to prove:
i. Specific intent to assist felon avoid arrest or escape,
ii. Aid is personally given to felon (merely failing to report is not enough)
iii. The person who was harbored/concealed/aided must have actually committed a
felony.
c. Sentence: relative felony

II. DEFENSES
A. Infancy
1. D will be exempt from criminal responsibility if younger than 10 years old.
2. If under age 17, it will be an act of juvenile delinquency, with some exceptions for teenagers ages
15-16, who may be tried in the adult courts.
B. Insanity
1. If D is incapable of distinguishing right from wrong in regard to the conduct in question
because of a mental disease or defect, he will be exempt from criminal responsibility.
2. D will not be exempt from criminal responsibility if he knows that his acts are condemned by
society.
3. Louisiana does not accept the doctrine of diminished responsibility. D may not introduce
evidence that because of mental disease or defect, he was incapable of forming the specific intent
required for the crime.
C. Intoxication
1. Applies to alcohol and drugs
2. Generally, intoxication at the time of the offense is immaterial unless:
a. If intoxication was involuntary and the direct cause of the criminal conduct, D will
be exempt from criminal responsibility; or
b. Negate existence of specific intent or special knowledge required in a particular crime.
D. Mistake
1. Of fact:
a. Reasonable ignorance or mistake of fact can be a defense unless precluded by the statute
(which it often is with sex crimes, etc).
b. Both a subjective (D actually made a mistake) and objective (the mistake was
reasonable) test.
2. Of law: ignorance of a law is not a defense, unless
a. D relies on the legislative repeal of a law which turns out to be ineffective; or
b. D relies on the final judgment of a court of last resort that a criminal law is
unconstitutional.
E. Justification
1. Justification defenses apply when:
a. D believes that his conduct is necessary under the circumstances; and
b. This belief is reasonable under the circumstances.
*Conduct = both need to use force and degree of force used
c. A person is generally only justified in using force to the degree apparently necessary to
accomplish his purpose.
2. Use of force is justified when:
a. Fulfillment of public duties/authorized by law (i.e. cop)
b. Discipline of minors
c. Failure to perform an affirmative duty because of physical impossibility
d. Circumstances of duress: commit any crime except murder
e. Protect another person when there is reason to believe that person could have used
the same means (includes killing)
3. Self defense: use of force that does not result in death is justifiable when committed to prevent:
a. Forcible offense against the person; or
b. Forcible offense or trespass against property in a person’s lawful possession.
4. Justifiable homicide (burden on state to prove the killing was not in self-defense):
a. In self-defense by a person who believes he is in imminent danger of death/GBH and
killing is necessary to save himself.
b. To prevent a violent or forcible felony involving danger to life or GBH. Intervention
must also create serious danger to the actor’s life.
c. Against a person who one believes is likely to use force against a person within a
dwelling or place of business while committing a burglary or robbery (or attempt).
d. By a person lawfully inside a dwelling or motor vehicle against a person attempting
to make unlawful entry or who has made unlawful entry, where person committing
homicide reasonably believes the deadly force is necessary to prevent entry or compel
the intruder to leave (carjacking/home invasion). Not applicable when person commits the
homicide is engaged in drug activity.
5. Aggressor cannot claim self-defense unless he withdraws and the other person knows or should
know that the danger is over.

III. INCHOATE OFFENSES


A. Conspiracy
1. Defined: agreement or combinations of two or more persons for the specific purpose of
committing any crime where at least one of the persons does an act in furtherance of the object of
the agreement/combination.
2. Elements
a. Agreement
i. Can be tacit or informal
b. Two or more people
i. Both need to actually agree  meeting of the minds
ii. Doesn’t need to be any direct communication, between all parties.
c. Agreement is for the specific purpose of committing a crime
d. Intent by each party
i. To agree; and
ii. To achieve a specific criminal objective
e. Act in furtherance of the conspiracy by at least one party
i. Does not have to be a criminal act
3. Can attribute acts/statements of one party during the course of and in furtherance of the
conspiracy to all co-conspirators.
4. Termination: ends with success or total frustration of the objective. D may withdraw by telling
the police or telling his co-conspirators.
B. Attempt
1. Elements
a. Specific intent to commit a crime
i. The crime that D wants to commit must actually be illegal
b. Doing or omitting an act
c. For the purpose of and tending directly toward the accomplishment of his object.
i. Mere preparation does not constitute attempt
ii. Being able to actually accomplish the object is immaterial.
C. Inciting a felony:
1. Endeavor by one person
2. To incite or procure another person
3. To commit a felony
D. Solicitation for Murder
1. Intentional solicitation by one person of another person to commit or cause to be committed
first- or second-degree murder.

IV. HOMICIDE
A. First-degree murder
1. Elements
a. Causing the death of another human being, when D has
b. Specific intent to kill or inflict great bodily harm; and
c. One of the following triggering factors:
i. D committed the murder while committing or attempting to commit:
a. Agg. rape h. Second-degree robbery
b. Forcible rape i. First-degree robbery
c. Agg. kidnapping j. Simple robbery
d. Second-degree kidnapping k. Cruelty to juvenile
e. Agg. arson l. Terrorism
f. Agg. escape j. Agg. burglary
g. Armed robbery k. Assault by drive-by shooting
ii. D had specific intent to kill two or more people
iii. V was under 12 years or over 65 years of age
iv. D was engaged in drug sale/distribution
v. D was participating in ritual ceremony
vi. Murder for hire
vii. Murder of witness or witness’s immediate family member
viii. V is peace officer engaged in performance of his duty
ix. D has previously committed a crime with specific intent to kill or inflict great
bodily harm that has resulted in the death of one or more persons.
x. Is violating a no-contact order between D and V.
xi. V is a taxi driver
Under 12 or over 65

Special V Witness or witness’s immediate family


characteristic  member
The killing of a Taxi driver
FIRST- human being with
Peace officer engaged in performance
DEGREE IS the specific intent to PLUS
of duty
MURDER kill or inflict great
bodily harm Specific intent to kill more than one
person
Committing or attempting to commit
enumerated felony
Violating no-contact order between
D’s behavior 
him and V
Murder for hire
Participating in ritual ceremony
Engaged in drug deal
Has previously committed a crime with
specific intent to kill or inflict great
bodily harm that resulted in the death
of one or more people
B. Second-degree murder
1. Elements: Killing of a human being,
a. When D has a specific intent to kill or inflict great bodily harm; OR
b. When D is engaged in the perpetration or attempt of one of the following felonies:
a. Agg. rape h. Second-degree robbery
b. Forcible rape i. First-degree robbery
c. Agg. kidnapping j. Simple robbery
d. Second-degree kidnapping k. Cruelty to juvenile
e. Agg. arson l. Assault by drive-by shooting
f. Agg. escape j. Agg. burglary
g. Armed robbery
Even though he has no intent to kill or inflict great bodily harm; OR
c. When D unlawfully distributes or dispenses a controlled dangerous substance or
gives it to another person to distribute/dispense, and this is the direct cause of V’s death;
OR
d. When D is engaged in perpetration of cruelty to juveniles even though he has no intent
to kill or inflict great bodily harm.
C. Attempted second-degree murder
1. The above elements, but
2. MUST PROVE SPECIFIC INTENT TO KILL
D. Manslaughter (felony)
1. Homicide which would be either first- or second-degree murder, BUT the offense is committed
a. In sudden passion or heat of blood immediately
b. Caused by provocation,
c. Which would deprive an average person of his self-control and cool reflection
*If jury finds his blood actually cooled or a reasonable person’s blood would have cooled, it
isn’t manslaughter.
2. A homicide committed without any intent to cause death or great bodily harm,
a. Where D is engaged in the perpetration or attempted perpetration of any
i. Felony besides the ones enumerated above, or
ii. Intentional misdemeanor directly affecting the person
b. Where D is resisting lawful arrest by means or in a manner not inherently
dangerous, and the circumstances are such that the killing would not be first- or
second-degree murder.
E. Negligent homicide (relative felony)
1. Either
a. The killing of a human being by criminal negligence; or
b. The killing of a human being by a dog or other animal where the owner is reckless
and criminally negligent in confining or restraining the dog or other animal. Unless it’s a
service animal.
F. Vehicular homicide (relative felony)
1. Elements
a. Killing of a human being,
b. Proximately or directly caused
c. By an offender engaged in the operation of or in actual physical control of any motor
vehicle, aircraft, vessel, watercraft, or other means of conveyance,
d. Whether or not D has the intent to cause death or great bodily harm, and
e. D is under the influence of drugs or alcohol.
2. If D is not under the influence of drugs or alcohol, this would be negligent homicide.

V. BATTERY/ASSAULT
A. Battery
BATTERY PLUS… EQUALS…
Dangerous weapon Aggravated battery
Intentional use of force or Intentional infliction of serious Second-degree battery
violence against the person of bodily injury
another, without V’s consent. Intentional infliction of serious Aggravated second-degree battery.
bodily injury WITH a dangerous
weapon
1. Definition:
a. Intentional use of force or violence against the person of another; or administration of
poison or other noxious liquid or substance to another.
c. Mens rea: general intent, with the exception of second-degree battery.
d. Prosecutor must prove V’s lack of consent.
2. Elements
a. D applies some physical force toward V’s person
b. The force is unlawful
3. Aggravated battery
a. Battery committed with a dangerous weapon
4. Second-degree battery
a. Battery
b. Where D intentionally
c. Inflicts serious bodily injury
i. Unconsciousness
ii. Extreme physical pain
iii. Protracted and obvious disfigurement
iv. Protracted loss/impairment of a bodily member/organ or mental faculty.
v. Substantial risk of death
5. Aggravated second-degree battery
a. Battery
b. Committed with a dangerous weapon
c. Where D intentionally inflicts serious bodily injury
6. Simple battery:
a. Battery committed without V’s consent
7. Special battery statutes
a. Battery of a police officer: Battery where V is a police officer and D has reasonably
grounds to believe that V is a police officer acting in the performance of his duty.
b. Battery of school teacher
c. Battery of child welfare worker
d. Battery of referee
e. Battery of infirm
f. Domestic abuse battery
g. Battery of correctional officer
B. Negligent Injuring
1. Injuring another
2. Through criminal negligence; OR
3. The inflicting of any injury by a dog or other animal where D was reckless and criminally
negligent in confining that animal.
C. Vehicular negligent injuring
1. Elements
a. Inflicting of any injury upon the person of a human being
b. When proximately or directly caused by D engaged in the operation or actual physical
control of a vehicle, aircraft, or boat
c. While D is under the influence of drugs or alcohol
2. First-degree vehicular negligent injuring:
a. Inflicting of any injury upon the person of a human being
b. When proximately or directly caused by D engaged in the operation or actual physical
control of a vehicle, aircraft, or boat
c. While D is under the influence of drugs or alcohol
d. Where D inflicts serious bodily injury.
D. Assault
SIMPLE
ASSAULT: Firearm Aggravated
Assault with
Attempt to commit firearm
a battery

OR Other dangerous Aggravated


USING weapon EQUALS Assault
Placing V in
reasonable
apprehension of Firearm and use of Assault by drive-by
receiving a battery motor vehicle to shooting
facilitate

1. Simple Assault:
a. The attempt to commit a battery; or
b. The intentional placing of another in reasonable apprehension of receiving a battery.
i. Reasonable = D must have apparent present ability to actually inflict that battery.
2. Aggravated Assault
a. Assault
b. Committed with a dangerous weapon
3. Aggravated Assault with a Firearm
a. Assault
b. Committed with a firearm
4. Special Assault statutes
a. Assault by drive-by shooting
i. Assault
ii. Committed with a firearm
iii. Where D uses motor vehicle to facilitate the assault
b. Aggravated assault on a peace officer with a firearm
i. P/O acting in course & scope of duty
ii. No discharge required
c. Aggravated assault with a motor vehicle upon a peace officer
i. P/O acting in course and scope of duty
d. Assault on school teacher
e. Mingling harmful substances
f. Assault on school teacher
g. Assault on utility worker with gun
h. Use of laser on police officer
E. Harassment crimes
1. Stalking
a. Intentional and repeated
b. Following or harassing of another person
c. That would cause a reasonable person to feel alarm or emotional distress.
*Exception for private investigators
2. Cyberstalking: use of email to extort, threaten, or harass.
3. Terrorizing
a. Intentional communication of information
b. That the commission of a crime of violence is imminent or in progress or that a
circumstance dangerous to human life exists or is about to exist, and
c. With the intent of causing members of the general public to be in sustained fear for their
safety; or causing evacuation of a building, public structure, or facility of transportation; or
causing other serious disruption to the general public.
4. Cyberbullying
a. V must be under 18
b. Includes any electronic communication: textual, visual, written, oral.
5. Terrorism:
a. D has intent
i. To intimidate or coerce the civilian population,
ii. Influence the policy of a unit of government by intimidation or coercion, or
iii. Affect the conduct of a unit of government by intimidation or coercion;
b. By the commission of:
i. Intentional killing,
ii. Intentional infliction of serious bodily injury,
iii. Kidnapping,
iv. Aggravated arson, or
vi. Intentional aggravated criminal damage to property.
6. Intimidation by police officer
a. Use of threats or violence to get a confession from someone in custody
F. Kidnapping
1. Agg. kidnapping (ransom):
a. Either
i. Forcible seizing & carrying of V from one place to another,
ii. Enticing or persuading V to go from one place to another, or
iii. Imprisoning or forcible secreting of V,
b. With the intent to force V or some other person to give up anything of apparent
present or prospective value or to grant any advantage or immunity
c. In order to secure a release of V.
2. Second-degree kidnapping
a. Either
i. Forcible seizing & carrying of V from one place to another,
ii. Enticing or persuading V to go from one place to another, or
iii. Imprisoning or forcible secreting of V,
b. Where V is:
i. Used as a hostage or shield,
ii. Used to facilitate the commission of a felony or a flight after attempt, or
iii. Physically injured or sexually abused.
3. Simple kidnapping
a. Intentional and forcible seizing and carrying of any person from one place to another
without his consent; or
b. Intentional taking, enticing, or decoying away for any unlawful purpose any child not D’s
own and under 14 years of age without consent of parent/custodian; or
c. Intentional taking, enticing, or decoying away and removing from the state by any parent
his or her child without the consent of the legal custodian in an attempt to defeat custody, or
d. Intentional taking, enticing, or decoying away any person committed to an institution
without the consent of the proper authority; or
e. Taking, enticing, or decoying away and removing from the state, by any person other than
the parent, a child placed in his or her custody by any court of competent jurisdiction with
an intent to defeat that court’s jurisdiction in awarding custody.
4. Aggravated kidnapping of a child: unauthorized taking, enticing, or decoying from a location for
unlawful purposes a child under age 13 with intent to secret him from parent/guardian.
a. Does not apply to parent, grandparent, legal guardian.
G. False Imprisonment
1. Elements
a. Intentional confinement or detention of another
b. Without V’s consent or legal authority
2. False imprisonment with dangerous weapon  like above, but D is armed w/ dangerous
weapon

VI. SEXUALLY BASED OFFENSES (ARE CONSIDERED ESPECIALLY HEINOUS)


A. Rape
1. Rape defined:
a. Anal, oral, or vaginal sexual intercourse
b. With a male or female person
c. Without that person’s lawful consent
2. Simple rape: rape where the sexual intercourse is deemed to be without V’s lawful consent
because
a. V incapable of resisting or understanding the nature of the act because of stupor or
abnormal condition of mind caused by narcotic or anesthetic or other CDS where D
knew or should have known of V’s incapacity
b. V, through unsoundness of mind, is temporarily or permanently incapable of
understanding the act and D knew or should have known of V’s incapacity
c. V submits to sex with someone she thinks is her husband and this belief is
intentionally induced through D’s fraud, etc.
3. Forcible rape: rape where the sexual intercourse is deemed to be without V’s lawful consent
because
a. V is prevented from resisting the act by force or threats of physical violence under
circumstances where V reasonably believes that such resistance would not prevent this rape.
b. V is incapable of resisting or understanding the nature of the act because of stupor or
abnormal condition of mind caused by narcotic or anesthetic or other CDS administered by
D w/out V’s knowledge.
4. Aggravated rape: rape where the sexual intercourse is deemed to be without V’s lawful consent
because
a. V resists to the utmost, but the resistance is overcome by force.
b. V is prevented from resisting by threats of great/immediate bodily harm accompanied by
apparent power of execution
c. V is prevented from resisting because D is armed with a dangerous weapon
d. V is under the age of 13 or over the age of 65
e. 2 or more offenders participate (commit or physically assist with) in the act
f. V is prevented from resisting because V suffers from physical or mental infirmity
preventing resistance
B. Sexual Battery  no penetration

VII. MISAPPROPRIATION CRIMES


A. Theft
1. Elements
a. Misappropriation or taking
b. Of anything of value
c. Which belongs to another
d. Either
i. Without the consent of the person; or
ii. By means of fraudulent conduct, practices, or representation.
2. Notes
a. Can include a taking by one in lawful possession.
B. Identity Theft:
1. Intentional use/attempted use
2. Of personal identifying information (SSN, driver’s license number, etc)
3. With fraudulent intent
4. To obtain anything of value
5. Without consent or authorization
C. Computer Fraud
1. Accessing or causing to be accessed of any computer system, computer network, or any part
thereof,
2. With intent to
a. Defraud; or
b. Obtain money, property, or services by means of false or fraudulent conduct, practices, or
representations.
D. Theft by Unauthorized use of Access Card
1. Elements
a. Use of access card = any means to access account
i. SSN, password, etc.
b. Used to obtain anything of value, whether contemporaneously or not
c. With intent to defraud
2. Includes the following acts
a. Use of forged access card
b. Making a reference to a nonexistent access card
c. Stealing/wrongfully appropriating an access card
d. Using an access card belonging to another w/out authority
e. Using a revoked access card
3. Access card itself is a thing of value ($100)
E. Unauthorized Use of Movable
1. Elements
a. Intentional taking or use of a movable
b. Which belongs to another
c. Either
i. Without the other’s consent; or
ii. By means of fraudulent practices, conduct, or representations.
d. But without any intention to permanently deprive the other of the movable
F. Unauthorized Use of a Motor Vehicle
1. Intentional taking
2. Of a motor vehicle belonging to another
3. Either
a. Without authority; or
b. By fraud
G. Worthless Check Statute
1. Issuing a check, draft, or order
2. To obtain something of value
3. Knowing that there are insufficient funds for payment
4. With intent to defraud
H. Illegal possession of stolen things
1. Elements
a. Intentional possessing, procuring, receiving, or concealing
b. Of anything of value
c. Which has been the subject of any robbery or theft
d. Under circumstances which indicate that D knows or has reason to believe that the thing
was subject to one of these offenses
2. Notes
a. Affirmative defense: within 72 hours of acquiring actual/constructive knowledge, writes
to the DA that the thing was stolen
b. Mens rea: intentionally with knowledge  excludes negligent or accidental acquisition.
I. Illegal Possession of Stolen Firearm
1. Intentionally possessing, procuring, receiving, or concealing
2. Of a firearm
3. That D knew or should have known was the subject of a robbery or theft
J. Purse Snatching
1. Theft of anything of value
2. Contained w/in a purse or wallet
3. From the person of another or within the immediate control of another
4. By use of force, intimidation, or by snatching,
5. But not while armed w/ a dangerous weapon
K. Theft of utility services
1. Electricity, gas, water, or telecommunications

VIII. ROBBERY
A. Armed Robbery
1. Taking of anything of value
2. Belonging to another
3. From
a. The person of another
b. The immediate control of another
4. By use of
a. Force
b. Intimidation
5. While armed with a dangerous weapon
B. First-degree Robbery
1. Taking of anything of value
2. Belonging to another
3. From
a. The person of another
b. The immediate control of another
4. By use of
a. Force
b. Intimidation
5. When D leads V to believe that he is armed with a dangerous weapon (this belief must be
both subjective and objectively reasonable)
C. Second-degree Robbery
1. Taking of anything of value
2. Belonging to another
3. From
a. The person of another
b. The immediate control of another
4. By use of
a. Force
b. Intimidation
5. When D intentionally inflicts serious bodily harm.
D. Simple Robbery
1. Taking of anything of value
2. Belonging to another
3. From
a. The person of another
b. The immediate control of another
4. By use of
a. Force
b. Intimidation
5. Where D is not armed with a dangerous weapon

SIMPLE ROBBERY: Dangerous weapon Armed Robbery


Taking of anything of value
belonging to another from Intentionally
the person of another or the PLUS inflicting serious IS Second-degree robbery
immediate control of bodily harm
another by use of force or Leading V to believe
intimidation. that D is armed with First-degree robbery
a dangerous weapon
E. Notes
1. Taking must be from V’s person or control  this depends on proximity and control. This is an
objective inquiry  V trapped in next room is probably fine whereas V being miles away probably
isn’t.
2. Force does not include surreptitious pickpocketing
3. The threat must be of immediate harm, not future harm.
4. Intimidation must be one which excites reasonable apprehension of danger  injury to V, other
persons.
5. Taking may precede the force/violence if the force/violence prevents the immediate recapture of
the property
F. Carjacking
1. Intentional taking
2. Of a motor vehicle belonging to another person
3. In the presence of that person or in the presence of a passenger or other person in lawful
possession of the vehicle
4. By force or intimidation
G. Extortion
1. Elements
a. Communication of threats to another
b. With the intention
c. To obtain anything of value or any acquittance, advantance, or immunity
2. Notes
a. Not necessary that D actually obtain something from V
b. Threat here (unlike the threat in robbery statutes) is one of future harm

IX. BURGLARY
A. Aggravated burglary
1. Elements
a. Unauthorized entering
b. Of any
i. Inhabited dwelling, or
ii. Structure, watercraft, or movable where a person is present
c. With the intent to commit a felony or any theft therein
d. If D
i. Is armed w/ a dangerous weapon;
ii. Arms himself with a dangerous weapon after entering; or
iii. Commits a battery upon any person while in the place or while entering or
leaving the place.
2. Notes
a. Commission of the theft/felony is not necessary, just intent.
b. Must have intent at the time entry happens  if D forms the intent while inside, it is
not burglary
c. If the place is open to the public, the authorization to enter is implied if D enters while it
is open to the public and remains in an area which is open to the public.
B. Simple Burglary
1. Elements
a. Unauthorized entering
b. Of any dwelling, structure, watercraft, or cemetery, movable, or immovable where a
person is present
c. With the intent to commit a felony or any theft therein
2. Note: this statute is broad enough to include any burglary that is not aggravated
C. Simple Burglary of an Inhabited Dwelling
1. Unauthorized entering
2. Of any inhabited dwelling, house, apartment, or other structure
3. Used in whole or part as a home or place of abode by one or more persons
4. With the intent to commit a felony or any theft therein
D. Unauthorized Entry of Inhabited Dwelling
1. Intentional entering
2. Of any inhabited dwelling or other structure belonging to another
3. Used in whole or in part as a home or place of abode by a person
4. By a person without authorization
E. Unauthorized Entry of a Dwelling During Emergency or Disaster  unless attempting to rescue or seek
refuge
F. Unauthorized Entry of a Place of Business
1. Intentional Entry w/out authority of any structure or premises belonging to another that is
completely enclosed by any type of physical barrier that is at least six feet in height and used in
whole or in part as a place of business.
G. Looting
1. Entry into any building, structure, etc.
2. Where normal security is not present due to a hurricane, flood, fire, or force majeure
3. By virtue of riot, mob, or human agency
4. And the obtaining of control over or damaging or removing property of the owner.
H. Home Invasion
1. Unauthorized entry
2. Of structure used as home
3. Where a person is present
4. With specific intent to
a. Use force against person; or
b. Damage property
I. Criminal Trespass
1. Unauthorized entry
2. Into any structure, watercraft, or movable
3. Or intentional entry onto immovable property owned by another.
J. Entry on or remaining in places or on land after being forbidden
1. Unauthorized going into or upon, remaining in or upon, or attempt to do so
2. On or in any movable or immovable of another
3. When forbidden orally or in writing, including by means of a sign
*Reasonably contemporaneous request to leave is an essential element

X. PROPERTY DAMAGE
A. Aggravated Arson
1. Intentional damaging
2. By
a. Any explosive substance; or
b. Setting a fire
3. Any structure, watercraft, or movable
4. Belonging to another
5. Where either of the following occur:
a. Any person suffers great bodily harm, permanent disability, or disfigurement as a
result of the fire or explosion; or
b. A firefighter, law enforcement officer, or first responder who is present at the scene
and acting in the line of duty is injured as a result of the fire or explosion.
B. Simple Arson
1. Scenario #1
a. Intentional damaging
b. By
i. Any explosive substance; or
ii. Setting a fire
c. Any property of another
d. Without the consent of the owner and except as provided in the aggravated arson
statute.
2. Scenario #2
a. Causing a fire, or
b. Starting an explosion
c. While D is engaged in
d. The perpetration of another felony offense
e. Even though D does not have the intent to start a fire or cause an explosion.
C. Arson W/ Intent to Defraud
1. Setting fire or damaging by any explosive substance,
2. Any property
3. With intent to defraud
D. Attempted Arson
1. Placing of combustible or explosive material
2. In or near any structure, watercraft, movable or forest land,
3. With the specific intent to eventually set fire.
E. Aggravated Criminal Damage to Property
1. Intentional
2. Damaging of structure, watercraft, or movable
3. When foreseeable that human life might be endangered
4. By means other than fire or explosion
F. Criminal Damage to Property
1. Intentional
2. Damaging of structure, watercraft, or movable
3. By means other than fire or explosion
G. Criminal Mischief: any of the following
1. Tampering with property of another without consent with intent to interfere with free enjoyment
of rights or intent to deprive of full use;
2. False fire alarm
3. Throwing stones at public places;
4. Communication to public utility or water services employee which places him in sustained fear
for his/another’s safety or results in an evacuation, when the communication is for the purpose of
disruption of service
5. Shooting a firearm at a train
6. False report to law enforcement officer of commission of a crime

XI. ESCAPE
A. Simple Escape
1. Intentional departure
2. Of a person imprisoned, committed, or detained
3. From
a. A place where such person is legally confined
b. Designated area of a place where such person is legally confined
c. Lawful custody of any law enforcement or DOC officer
4. Under circumstances where human life is not endangered
B. Aggravated Escape
1. Intentional departure
2. Of a person imprisoned, committed, or detained
3. From
a. A place where such person is legally confined
b. Designated area of a place where such person is legally confined
c. Lawful custody of any law enforcement or DOC officer
4. Under circumstances where human life is endangered

XII. CORRUPTION/DECEIT
A. Perjury
1. Elements
a. Intentional making of
b. A false written or oral statement
c. Made under oath or affirmation
d. In or for use in
i. A judicial;
ii. Legislative; or
iii. Committee proceeding
e. Relating to a matter material to the issue or question in controversy
2. Notes
a. Mens rea = knowingly; must know the statement is false or not know whether it’s true or
not and say it anyway.
b. There is a presumption of falsity if there is contradictory testimony in the same or other
proceeding, but D can rebut this.
B. Public Bribery
1. Giving or offering to give, directly or indirectly,
2. Anything of apparent present or prospective value
3. To any of the following persons
a. Public officer or employee
b. Person in a position of public authority
c. Election official at any general, special, or primary election
d. Grand or petit juror
e. Witness
f. Any person who has been elected to public office, regardless of whether or not they have
assumed title and started performing duties yet
4. With the intent to influence his conduct in relation to his position, employment, or duty; or
5. The acceptance or offer to accept by any of the above people, directly or indirectly, the subject of
the above offer.
C. Public Intimidation
1. Use of violence, force or threats
2. With intent to influence conduct in violation to his position, employment, or duty,
3. Upon any of the following
a. Public officer or employee
b. Grand or petit juror
c. Witness
d. Voter at public election
e. School bus operator
D. Obstruction of Justice
1. Act done w/ knowledge
2. That it has, may, or will affect a past, present, or future criminal proceeding
3. Includes the following acts
a. Tampering with evidence
b. Using or threatening force towards person or property to
i. Influence testimony or withholding of testimony
ii. Altering, destroying, or concealing evidence
iii. Delaying in reporting information to peace officer
c. Retaliating against W, V, juror, party, attorney, or informant
E. Compounding a Felony
1. Accepting something of value or the promise thereof
2. From a person who committed a crime
3. Upon agreement to withhold information from the authorities

XIII. GUN CRIMES


A. Unlawful sale of weapon to minor
B. Possession of a firearm or carrying a concealed weapon by a convicted felon
1. Enumerated felonies: murder, manslaughter, agg. battery, agg/simple rape, agg. kidnapping, agg.
arson, agg/simple burglary, armed/simple robbery, burglary of a pharmacy or inhabited dwelling,
sex offense, any felony drug crime.
2. Includes attempt.
C. Illegal use of weapon
1. Intentional or criminally negligent
a. Discharge or any firearm, or
b. Throwing, placing, or other use or any article, liquid, or substance
2. Where it is foreseeable that it may result in death or great bodily harm to a human being.
D. Illegal carrying of weapon
1. Either
a. Intentional concealment on one’s person of a firearm or other instrumentality
customarily used or intended for probable use as a dangerous weapon;
b. Ownership, possession, custody, or use of any tools, explosives, nitroglycerin, or
instrumentality customarily used by thieves or burglars;
c. Manufacture, ownership, possession, custody, or use of a switchblade or spring knife or
similar instrument; or
d. Intentional possession or use of dangerous weapon on school campus during regular
school hours or on a school bus (exempts law enforcement officers or school officials acting
in course of their duty)
E. Illegally Supplying a Felon With a Firearm
F. Places where you cannot carry a firearm (“Illegal Carrying of Firearm at/on _____”)
1. School property
2. Parade
3. Alcoholic beverage outlet

XIV. OFFENSES AFFECTING LAW ENFORCEMENT


A. Resisting an Officer
1. Either
a. Intentional interference, opposition, resistance, or obstruction or a person acting in an
official capacity to arrest, search/seize, or serving process/court order, knowing or having
reason to know that the person is acting officially.
b. Flight by one sought to be arrested after notice of arrest
c. Violence towards arresting officer
d. Refusal to identify self or providing false information about identity
e. Refusal to move on when ordered by an officer.
B. Flight from an Officer
1. Refusing to stop vehicle (by driver of vehicle)
2. Knowing that there has been visual or audible signal to stop by a police officer who has
reasonable grounds to believe driver has committed offense.
C. Aggravated Flight from an Officer
1. Refusing to stop vehicle after signal to do so
2. And endangering human life in the process
a. Collision
b. Driving 25 miles over speed limit
c. Driving against traffic flow
d. Leaving roadway or forcing another to do so
e. Failure to obey traffic light or stop/yield sign
D. Interfering w/ Law Enforcement Investigation
1. Intentional obstruction or interference
2. Of a law enforcement officer conducting investigative work at a crime scene or accident
3. By refusing to move or leave when ordered to do so
4. When D has reasonable grounds to believe that the officer is acting in furtherance of his official
duties.

XV. DRUG CRIMES


A. Manufacturing
B. Distributing
C. PWIT

XVI. MISCELLANEOUS
A. Operating a Vehicle While Intoxicated
1. Applies to any motor vehicle, aircraft, boat, etc.
2. Applies when D’s BAC is more than .08 or he is under the influence of illegal drugs or
prescription drugs.
B. Disturbing the Peace
1. Doing any of the following in a manner which would foreseeably disturb or alarm the public
a. Fighting
b. Use of derisive words/name calling
c. Appearing in intoxicated condition
d. Engaging in any act in a violent or tumultuous manner by 3 or more persons
e. Unlawful assembly
f. Interrupting lawful assembly

CONSTITUTIONAL CRIMINAL PROCEDURE

I. EXCLUSIONARY RULE
A. General
1. Prohibits the introduction of evidence that was obtained in violation of D’s 4th, 5th, or 6th
Amendment rights.
2. This is the remedy, not the right it protects.
B. Scope
1. Fruit of the poisonous tree  the following will be excluded:
a. Any illegally obtained evidence
b. Any evidence derived from that evidence
2. Fruits derived from Miranda violations may still be admissible.
3. Balancing test: SCOTUS has emphasized that lower courts should balance the rule’s purpose
(deterrence) against its costs (exclusion of probative evidence) when deciding whether or not to
apply the exclusionary rule.
3. Exceptions
a. Independent source: evidence is admissible if the prosecution can show that it was
obtained from a source independent of the original illegal conduct
i. i.e. Police search your house w/out a warrant, see some drugs, and leave to get a
warrant, then come back.
b. Attenuation: intervening act of free will by D will break the causal chain between the
evidence and original illegality.
c. Inevitable discovery: If state can show that the police would have discovered the
evidence whether or not they acted unconstitutionally, the evidence is admissible.
d. Live witness testimony: difficult for D to get this excluded because it’s hard to prove
that a sufficiently direct link exists.
e. In-court ID: D may not exclude this.
4. Limitations
a. Grand Jury
b. Police officer acting in good faith: relying on a good faith warrant/law when they arrest
or search someone. Exceptions:
i. Affidavit is so lacking in PC that no reasonably P/O would believe it.
ii. Warrant defective on its face
iii. P/O lied or misled magistrate
iv. Magistrate “wholly abandoned his judicial role.”
c. Excluded evidence may be used to impeach D.
i. Confessions obtained in violation of Miranda
ii. Evidence seized pursuant to an illegal search
d. Knock & announce rule: exclusion is not an available remedy for this.
C. Standard of Appellate Review
1. Harmless error analysis  if app. court determines beyond a reasonable doubt that the evidence
did not affect the outcome, the conviction will stand.
D. Enforcing the Exclusionary Rule
1. D files motion to suppress  state must prove by a preponderance of the evidence that the
evidence is admissible.
2. D has right to testify at the hearing with the evidence being used against him at trial
II. FOURTH AMENDMENT
A. General
1. Fourth Amendment provides that people should be free from unreasonable searches and seizures.
2. Search = govt. intrusion into an area where a person has a reasonable/justifiable expectation of
privacy.
3. Seizure = exercise of control by govt. over a person or thing.
a. Seizure of a person: P/O’s physical application of force OR D’s submission to P/O’s show
of force.
i. Given the totality of the circumstances, a reasonable person would feel that he
was not free to terminate the encounter.
ii. Police pursuit is not enough.
b. On exam: “explain officer’s justification for pursuing/detaining D”  wants you to
explain the probable cause/reasonable suspicion
4. Reasonable = totality of the circumstances test
B. Arrests
1. When D is taken against his will into custody for purposes of criminal prosecution or
interrogation.
2. Probable cause requirement
a. The Fourth Amendment requires that all arrests be based upon probable cause.
b. PC = reasonably prudent person would believe that D committed or is committing a
crime.
c. Mistakes about the offense D committed is okay as long as P/O had PC for something.
3. Warrant requirement
a. No warrant req. in public place
i. Felony arrest = reasonable grounds to believe that a felony was committed by D.
ii. Misdemeanor = may arrest for misdemeanor committed in P/O’s presence.
b. P/O needs warrant when making an arrest in the home of D or a third party.
i. Regarding evidence seized at a third party’s home, D will likely not be able to get
it excluded unless he can establish REOP at the home  i.e. overnight guest.
4. Effect of warrantless arrest: potential exclusion of any resulting evidence, but not effect on
prosecution.
C. Lesser detentions
1. Stop & frisk
a. P/O has authority to briefly detain D for investigative purposes even if they lack PC to
arrest. They need a “reasonable suspicion” supported by “articulable facts” of criminal
activity.
i. Reasonable suspicion is less than PC, but more than a vague suspicion.
ii. Reasonable suspicion does not have to arise from P/O’s personal knowledge.
b. If P/O also has reasonable suspicion to believe that D is armed and dangerous, he may
conduct a frisk for weapons.
c. No set time limit, but P/O must act in a diligent and reasonable manner in
confirming/dispelling their suspicions.
d. It only constitutes a stop if there’s detention.
2. Vehicle stop
a. P/O may not stop a car unless he has at least reasonable suspicion to believe that a law
has been violated.
b. Exception for special law enforcement needs where the roadblocks/stops are based on a
neutral, articulable standard and are designed to serve purposes closely related to a
particular problem pertaining to cars and then mobility (i.e. drunk driving, illegal immigrant
checkpoint, etc).
i. Police may not set up roadblocks to check cars for illegal drugs.
ii. Informational roadblocks are okay: i.e. asking everyone if they have any
information about kidnapping.
c. Pretextual stops: okay, because you measure the objective circumstances, not P/O’s
subjective thoughts (brake tag example).
d. Seizure
i. Car stop constitutes seizure of driver AND occupants
ii. Once a vehicle has been lawfully stopped, police may order driver and passengers
out.
iii. If driver reasonably believes that driver or passenger(s) may be armed and
dangerous, they may conduct a frisk.
iv. Can search passenger compartment of a vehicle for weapons once everyone has
been ordered out.
3. Detention to obtain a warrant: allowed if police have PC to believe that D has hidden drugs
inside home
a. Can detain both D and other occupants of the premises
4. Station house detention: need full PC
5. Grand jury subpoena: does not constitute a seizure
6. Deadly force: this is a seizure. P/O may not use deadly force unless it is reasonable under the
circumstances (broadly interpreted).
D. Evidentiary search & seizure
1. Exam analysis
a. Did D have a 4th Amendment right?
i. Was there govt. conduct?
ii. Did D have REOP?
b. Did police have a valid warrant?
c. If not, did they make a valid warrantless search and seizure?
2. Requirements for the 4th amendment to apply
a. Govt. conduct  4th Am. does not apply to private actors
b. D has REOP in the place where the thing was seized
i. No REOP for things held out to be public: your clothes, paint on outside of car,
smell of your luggage, bank records, movement of car on public roads, etc.
ii. Squeezing luggage: REOP
iii. Dog sniffs at lawful traffic stops: okay
iv. Open fields doctrine: No REOP in areas outside the curtilage of your home.
v. Fly-overs with naked eye: okay
vi. Technologically-enhanced searches of home: not okay, especially if the
technology isn’t in general public use (i.e. thermal imaging).
3. Requirement for a search to be valid under the 4th Amendment: be made pursuant to a valid
warrant. To be valid, a warrant must be:
a. Issued by neutral & detached magistrate
b. Based on PC that seizable evidence will be found on the premises or person to be
searched.
i. Anticipatory warrant okay
ii. CI tip: totality test to determine sufficiency of information
c. Particularly describe place to be searched and things to be seized
i. Example: duplex that police thought was only one apartment
ii. Listing of categories of evidence (i.e. guns, fruits, instrumentalities) will probably
be okay, especially if it’s a complex crime.
4. Attacking a search warrant  must meet 3 requirements to invalidate a valid warrant (Franks
hearing)
a. False statement in affidavit
b. Made intentionally or recklessly by affiant
c. False statement was material to the finding of probable cause
5. Evidence seized pursuant to invalid warrant (no PC) may be admissible if it were obtained by the
police officer in reasonably reliance on a facially valid warrant.
6. Execution of warrant
i. Executed by police
ii. Executed w/out unreasonable delay
iii. Knock & announce
a. Unless police reasonably believe that doing so would inhibit the investigation
iv. Scope of search: limited to what is reasonably necessary to discover the items listed in
the warrant.
a. Police may seize anything they find regardless of whether it is in the warrant,
v. A search warrant does not authorize officers to search people found on the premises, but
they can search them incident to arrest if they have PC to arrest them.
vi. Executing a SW carries with it the power to detain occupants while a proper search is
conducted.
E. Exceptions to Warrant Requirement
1. Plain View
a. Police may make a warrantless seizure when:
i. They are lawfully on the premises;
ii. The evidence/contraband is in plain view; and
iii. Its incriminating character is immediately apparent (PC).
b. Moving stuff around to find out if it’s stolen (i.e. looking at serial numbers) means that
the incriminating character is not immediately apparent.
2. Consent
a. Police may conduct a valid warrantless search of a home if the owner consents that they
do so.
b. Scope of search is limited by the scope of the consent.
c. Co-owner may consent, unless the other co-owner is on the premises and protesting
against the search (Randolph).
i. This likely extends to apparent co-owners as well.
ii. Parent: can consent to search of child’s room, but the search may not extend to
locked containers  depends on child’s age.
d. Knowledge of right to w/hold consent is not a prerequisite to establishing consent.
e. Note: police officer’s lie that he has a warrant will negate any consent given.
3. Stop & Frisk
a. A police officer may stop and detain a person if he has reasonable suspicion to believe
that person committed a crime.
b. If the officer also has reasonable suspicion to believe that D is armed and presently
dangerous, the stop may involve a pat-down of the suspect’s outer clothing.
i. If the officer has specific information that D has a weapon somewhere, he may
reach directly and not just do a pat-down (i.e. belt).
ii. This also applies where police make a lawful traffic stop (which includes ordering
the passengers). If the officer reasonably believes that an occupant may be armed
and dangerous, he can frisk them. He may also search any areas in the vehicle where
a weapon may be placed or hidden, as long as he believes the occupant is armed and
dangerous. This can be done even if the officer doesn’t arrest the occupant.
c. No rigid time limit for the length of an investigative stop.
d. During a valid patdown, officer can seize any item that the officer reasonably believes is
contraband or drugs based on its “plain feel.”
4. Search Incident to Lawful Arrest
a. Police may conduct a warrantless search incident to arrest as long as the arrest was made
on PC.
i. Traffic stops do not count.
b. May be done:
i. In home
ii. Protective sweep (to find potential confederates). Buie.
iii. In car
iv. Pursuant to incarceration/impoundment
c. Geographic scope: D and D’s grabbing area
i. Grabbing area = any area where he might reach to obtain weapons or destroy
evidence. Grabbing area follows D as he moves.
i. Cars: can only search the interior if :
a. D is unsecured and still may gain access to the vehicle; or
b. Police reasonably believe that evidence of the offense for which D was
arrested may be found in the vehicle
d. The search must be contemporaneous w/ the arrest
i. For cars, if police have reason to believe the car contains evidence, they may first
move D to squad car and then search.
5. Exigent Circumstances/Emergency
a. Hot pursuit of fleeing felon: warrantless search as broad as necessary to capture the
suspect and prevent him from escaping. This includes pursuing him into private dwellings.
b. Emergency aid: police may effect a warrantless search if there is an emergency that
threatens health/safety if not immediately acted upon.
i. Seeing someone injured or threatened with injury
ii. Whether or not an emergency exists is determined objectively.
6. Automobile:
a. If police have PC to believe that a vehicle such as an automobile contains contraband or
fruits, instrumentalities, or evidence of a crime, they may search the vehicle w/out a warrant.
b. Scope:
i. If police have full PC to search a vehicle, they can search the entire vehicle –
including trunk – and all containers within it that might contain the object for which
they are searching.
ii. Includes passenger’s belongings.
c. Motor homes that are like cars (i.e. mobile) fall under the automobile exceptions.
d. Contemporaneous search not required  towing the vehicle and searching it later is OK
if the warrantless search is justified at the time the car is stopped.
7. Inventory: searching an arrestee’s personal belongings pursuant to arresting a defendant is okay.
8. Airline passengers, govt. employees, parolees: searches okay
9. School searches: no PC or warrant required, only reasonable grounds for the search are
necessary:
a. Offers moderate chance of finding evidence of wrongdoing;
b. Measures used are reasonably related to the objectives of the search;
c. Search is not excessively intrusive given suspect’s age and sex.
F. Due Process
1. Due Process requires that criminal prosecutions/investigations be conducted in a way that does
not offend the “sense of justice” inherent in due process  evidence obtained in a way that “shocks
the conscience” will be excluded.
2. Body searches: a blood sample is fine, whereas an invasive surgery is unreasonable
3. Confessions: D beaten, lied to, etc.

III. CONFESSIONS
A. Voluntariness
1. To be admissible, confession must be voluntary.
2. Only official compulsion will render a confession involuntary for purposes of the 14th
Amendment.
3. Standard of appellate review: harmless error test.
4. Excluded from impeachment as well
5. Even if confession is found to be voluntary, D may still present circumstances of the confession
to the jury  goes towards credibility.
B. Sixth Amendment Right to Counsel
1. When applicable
a. The 6th Amendment gives D the right to counsel at all critical stages of trial after
adversary criminal proceedings have been instituted.
i. Post-charge lineup
ii. Preliminary HG to determine PC to prosecute
iii. Post-indictment interrogation
iv. Arraignment
v. Felony trial
vi. Misdemeanor trials where imprisonment is imposed or suspended jail sentence
vii. Overnight recess during trial
viii. Guilty plea and sentencing
ix. Appeal as a matter of right
x. Appeals of guilty and nolo pleas.
b. Not applicable during:
a. Blood sampling
b. Taking of handwriting or voice samples
c. Pre-charge or investigative lineups
d. Photo lineup
e. Preliminary HG to determine PC to detain
f. Brief recesses during D’s testimony at trial
g. Discretionary appeal
h. Revocation proceeding
i. Post-conviction proceeding
th
c. (5 Amendment also gives D a right to counsel at all custodial police interrogations)
2. Offense specific: D must make 6th Amendment RTC request for each charge.
a. Blockburger test for different offenses used
3. Waiver: can waive 6th Amendment right to counsel. Waiver must be:
a. Knowing
b. Voluntary
c. Intelligent
4. Remedy:
a. RTC violated at trial: automatic reversal
b. Non-trial: harmless error test used
C. Fifth Amendment RTC  Miranda
1. General
a. The Fifth Amendment states that no D shall be compelled to be a witness against himself
 D cannot be compelled to give self-incriminating testimony.
b. Miranda warnings that must be given before a custodial interrogation
i. Right to remain silent
ii. Anything you say can and will be used against you in court.
iii. Right to an attorney
iv. If you cannot afford one, one can be appointed for you.
c. Warnings need not be verbatim
d. Generally, there is no need to repeat the warnings after a short break  for a prolonged
break, you probably should.
2. When required
a. Anyone in police custody and accused of a crime must be Miranda-ized prior to
interrogation.
b. Police = govt. actor, not a mall cop, private actor, informant who has not been instructed
by police to talk to D.
i. Applies to state-ordered psych evaluation as well
c. Custody = whether D is in custody depends on whether or not his freedom of action is
limited in a significant way.
i. Paradigm: stationhouse interrogation  the further away we get from this, the less
likely it is that D is in custody.
ii. Voluntary detention is not custody (i.e. routine traffic stop, D in own home and
not accused of crime).
iii. In contrast, a long and involuntary detention is custodial (D in jail on another
charge.
iv. Determined by the objective circumstances on a case-by-case basis.
v. Can take minor’s age into account if obvious to police (JDB)
d. Interrogation = express questioning, words or actions that police know are reasonably
likely to elicit an incriminating response from D.
i. Spontaneous statements, routine booking questions: not covered by Miranda.
ii. If D makes a spontaneous statement, police must Miranda-ize before questioning
further.
iii. If there is a break in interrogation and a second agency takes over, the impact of
an earlier denial of rights will carry over.
3. After Miranda warnings given:
a. D’s options
i. Do nothing and don’t answer: no waiver, but not an invocation either. Police may
continue to question D.
ii. Start talking in response to questions: waiver
iii. Invoke right to remain silent: police must scrupulously honor the invocation, but
may resume question if D reinitiates questioning.
a. D’s invocation must be:
1. Explicit
2. Unambiguous
3. Unequivocal
b. Whether or not police scrupulously honor the invocation depends on
totality of the circumstances.
1. i.e. whether or not police wait 5 minutes or 2 weeks to re-question
you.
iv. Invoke right to counsel: all questioning must cease until D gets a lawyer.
a. Invocation must be express: “I think I should see a lawyer” isn’t enough.
b. If D reinitiates questioning, police may interrogate and the confession
will be admissible.
c. Timeline: invocation of RTC valid for entire time D is in custody and 14
days after.
d. Can’t question D about anything, even unrelated crimes.
e. If D makes an ambiguous request, police may either ask for clarifications
or keep questioning him until they get an unambiguous waiver.
f. Invoke for written statement but not oral  oral statement will be
admissible.
g. Counsel must actually be present at the interrogation  consulting w/ him
beforehand isn’t enough.
b. Waiver must be:
i. Knowing
ii. Voluntary
iii. Intelligent
c. Question first/warn later technique: not okay if it was intentional. If both the original
questioning and the failure to warn seem inadvertent, the confession may be admissible.
d. Public safety exception to Miranda: if police interrogation is reasonably prompted by
concern for public safety, responses to the questions may be used in court (Quarles: D asked
where he’d hidden gun).

IV. PRETRIAL IDENTIFICATION


A. Types of ID
1. Live line-up
2. Photo array
3. Single photo
4. Show-up
5. Voice/smell/etc.
B. Substantive bases for attack
1. Violation of RTC:
a. D has the right to the presence of an attorney at any post-charge lineup or show-up. The
purpose is so the attorney can observe any suggestiveness and bring them out on cross at
trial.
b. This right does not extend to photo identifications or when the police take physical
evidence from D (handwriting, voice, etc).
2. Due Process violation:
a. Occurs when an identification is:
i. Unnecessarily suggestive; and
2. There is a substantial risk of misidentification.
b. Also occurs when the procedure is fundamentally unfair (D is the only black man in the
lineup, etc).
3. NO right to a lineup and NO basis to attack using the 5th Amendment, because there is no self-
incrimination/testimony.
C. Remedy
1. Exclusion of police officer’s testimony about the prior lineup; possibly exclusion of the in-court
ID at trial.
2. Exception: the in-court ID will not be excluded if the in-court ID has an independent source.
Court will weigh the following factors in determining if there’s an independent source:
a. W’s opportunity to observe D at the time of the crime
b. Ease w/ which the witness can ID the defendant.
c. Existence or absence of prior misidentifications.

V. PRE-TRIAL PROCEDURES
A. Gerstein Hearing
1. Preliminary hearing to determine probable cause to detain.
2. Applies where D is in custody or out on bail, but no PC has been determined (i.e. arrest pursuant
to GJ indictment).
3. Must be done w/in 48 hours.
4. No real remedy.
B. Rights Associated w/ Pre-Trial Detention
1. Soon after D is arrested, he must be brought before a magistrate, who will
a. Advise D of rights
b. Set bail (there is no right to bail, but there’s a right to be free from excessive bail)
c. Appoint counsel if necessary
C. Preliminary Hearing to Determine PC to Prosecute
1. RTC
2. Contradictory hearing  both sides can present evidence
3. D may waive hearing
4. Both sides can use hearing to preserve testimony of Ws for trial, as long as there is an
opportunity to cross-examine.
D. Grand Jury
1. No RTC or Miranda warnings
2. Attorney cannot be present but may consult with him outside
3. No right to “potential D” warnings
4. GJ may base indictment on inadmissible evidence
5. Generally no duty by state to disclose inadmissible evidence
E. Speedy Trial
1. Constitutional Right
a. Right comes from the 6th Amendment
b. Evaluation of whether or not D’s 6th Amendment rights have been violated is done by
considering the following factors:
i. Length of delay
ii. Reason for delay
iii. Whether D asserted his right to speedy trial
iv. Prejudice to D
c. Delays caused by D’s court-appointed lawyer should generally be attributed to D, not to
the state.
d. Remedy: dismissal with prejudice.
e. When right attaches: when D has been arrested or charged. Attaches even if D did not
know about the charges against him (Doggett).
f. Special problems
i. Detainees incarcerated in one jurisdiction with charge pending in another  have
a right to have the second jurisdiction exert reasonable efforts to get him there.
ii. Indefinite suspension of charges  violation of right to speedy trial to allow state
to indefinitely suspend charges, i.e. dismissing without prejudice and refiling
F. Brady Obligations
1. State has a duty to disclose material, exculpatory evidence to D.
2. Willful or inadvertent failure to do so is a violation of the Due Process Clause.
3. D’s conviction will be reversed if he can prove that:
a. The evidence is favorable to D (exculpatory or impeaching); and
b. Prejudice has resulted from it being withheld.
4. Only extends to material exculpatory evidence relevant to state’s case in chief.
5. D’s reciprocal duty to disclose:
a. Notice of alibi defense
i. State may not comment on D’s failure to produce W stated in the alibi notice but
may use the alibi notice itself to impeach if D testifies to different alibi
b. Insanity defense
G. Competency Determination
1. Depends on D’s mental condition at the current time (distinguished from insanity).
2. Due Process standard: D’s trial would be prohibited if he is incompetent to stand trial. This
happens either when:
a. D lacks a rational as well as factual understanding of the charges and proceedings; or
b. Lacks sufficient present ability to consult with his lawyer with a reasonably degree of
understanding.
3. Can be raised by either side or judge
4. Okay to put burden on D as long as it’s by a preponderance.
H. Pretrial Publicity & Right to a Fair Trial
1. May require change of venue or retrial

VI. TRIAL
A. Right to Trial By Jury
1. When the right applies
a. Any felony charge
b. Misdemeanor offense where imprisonment is authorized for more than 6 months
2. Number of jurors
a. First and second-class felonies: 12
b. Anything else: 6
c. Verdict:
a. No absolute right to unanimity, except for a 6-pack jury.
b. SCOTUS as upheld as low as 9-3.
3. Venire
a. Right to venire selected from represented cross-section of the community.
b. D may complain if a significant group has been excluded by showing the
underrepresentation of a group that is both:
i. Distinct; and
ii. Numerically significant
c. No right to proportionality on the petit jury  theoretically it could be all white men, etc.
4. Voir dire
a. Can’t use peremptory strikes in a discriminatory manner (Batson)
i. This applies to both prosecutors and defense attorneys
b. If opposing counsel does so, the proper avenue for relief is to raise a Batson challenge.
Three steps:
i. D must show facts and circumstances that raise an inference that the exclusion of
particular jurors was based on race or gender;
ii. Prosecutor can then offer a race-neutral reason for the strike; and
iii. Judge decides if the race-neutral explanation was genuine or if it was a pretext.
c. Prosecutors can also raise a Batson challenge.
5. Right to Impartial Jury
a. Right to questioning on racial bias
i. If inextricably bound to crime
ii. If not, only a capital D accused of an interracial crime is specifically entitled to
the questions (although many judges will allow it for Ds accused of less)
b. Death penalty: state and defense can strike jurors who are automatically opposed or
automatically believe in the death penalty (Witherspoon)
i. Only jurors whose views will prevent or substantially impair them from the
performance of their duties in accordance with the law will be struck.
ii. Death sentence imposed by a jury from which a juror was automatically excluded
is subject to automatic reversal.
B. Sentencing
1. Any fact, that if established will increase your mandatory minimum or maximum sentence, you
have the right to have that fact alleged in the indictment and found by a jury. Apprendi.
2. This also applies to sentencing enhancements found after pleading guilty.
3. Test: harmless error
4. However, it is at the judge’s discretion to decide whether sentences run concurrently or
consecutively.
C. Right to Counsel at Trial (6th Amendment)
1. Waiver: this right can be waived and D can represent himself if D waives knowingly and
intelligently and is found to be competent to proceed pro se  judge must have colloquy with D on
the record.
2. D who is not represented at trial and is convicted will get his conviction automatically reversed.
3. Effective Assistance of Counsel
1. Strickland test: D must prove both
a. Deficient performance; and
b. Prejudice to D because of attorney’s performance.
2. Trial tactics or failure to raise a constitutional claim that is later invalidated is NOT IAC.
3. Conflicts
a. D atty having a direct conflict of interest is IAC is D timely objects.
b. Conflict w/ attorney is rarely grounds for relief, unless D can demonstrate that the
conflict was so severe that the attorney could not effectively investigate or present
D’s claims.
c. No right to joint representation.
D. Right to Confront Witnesses
1. General
a. Absent an important public purpose to the contrary, D is entitled to a face-to-face
confrontation w/ Ws against him when they testify.
b. Right is not absolute:
i. D is disruptive
ii. D voluntarily leaves courtroom
iii. Govt. action discouraging D’s absence is not automatically a violation of this
right (state comments that by testifying last, D had chance to adjust and alter
testimony).
2. Bruton
a. Co-D’s statement that implicates D cannot be used at their joint trial if co-D does not
testify.
i. This is true even if it interlocks with D’s own confession, which is admissible.
ii. Can use the confession to rebut D’s assertion that his confession was obtained
coercively  jury must receive limiting instruction.
b. Proper remedy is generally to sever the two Ds and try them separately, as redaction
generally cannot be done effectively.
2. Crawford
a. Testimonial statements cannot be used at trial if the declarant is unavailable even if they
meet a hearsay exception, unless D had the opportunity to cross examine the declarant at the
time the statement was made.
b. Testimonial = statement given to law enforcement official for the purpose of use at trial,
etc. Statements where the primary purpose is to help the police respond to an ongoing
emergency. SCOTUS has not given an express definition, but at a minimum, the following
are included:
i. Testimony from preliminary hearing
ii. GJ testimony
iii. Statements made during a police interrogation
iv. Testimony from former trial
v. Results of forensic lab testing
c. D will forfeit Crawford rights if he kills V/W with the express purpose to prevent them
from testifying. Giles.
E. Burden of Proof
1. General: State must prove beyond a reasonable doubt
2. Related topic: presumption of innocence
F. Misc. Trial Rights
1. Right to public trial
2. Unbiased judge
3. Can’t force D to be tried in prison clothing
4. NO right to preservation of potentially exculpatory evidence – due process will only be violated
if the evidence is destroyed in bath faith (Youngblood)

VII. GUILTY PLEAS & SENTENCING


A. Effect of GP
1. Waives right to jury trial
B. Procedure
1. Judge must determine that the plea is
a. Voluntary; and
b. Intelligent.
2. This is done by addressing D personally in open court on the record.
3. Judge must be sure D understands things such as:
a. Maximum possible penalty/mandatory minimum
b. That he has a right not to plead guilty
c. That he waives his right to trial by pleading guilty
4. A defendant who has not been properly informed is not bound by the plea
5. Remedy is withdrawl of the plea and pleading anew.
6. No requirement that the record contain the factual basis for the plea or evidence of D’s guilt.
C. Bases for Attack of Plea After Sentence
1. Involuntariness
2. Lack of jurisdiction
a. Court who took it lacked jurisdiction
b. Charges were barred because of double jeopardy
3. IAC
4. Failure by state to keep up their end of the plea bargain
D. Plea Bargaining
1. D who enters into a plea bargain has a right to have that bargain kept. Plea bargain will be
enforced against D and state, but not against the judge.
a. If state does not keep their bargain, the court will determine whether specific performance
is required or if D should be allowed to withdraw plea.
b. If D does not keep his end of the bargain, his plea and sentence can be vacated.
2. State has the power to threaten a more serious charge; as well as the power to charge the more
serious offense
3. Statements made by D in the course of unsuccessful plea bargaining are inadmissible, unless D
knowingly and voluntarily waives this protection.
4. D not entitled to impeachment or affirmative defense evidence before he pleads.
D. Sentencing
1. 8th Amendment protection against cruel and unusual punishment
a. Punishment that is grossly disproportionate to the offense is cruel and unusual (SCOTUS
does not find this often, even for lengthy sentences for habitual offenders of petty crimes).
b. No right to comparison of penalties in similar cases.
2. Recidivist statutes: mandatory life sentence imposed pursuant to a recidivist statute is not
unconstitutional even if the three prior felonies were non-violent and property-related.
3. Judge may take into account D’s perjury at trial when deciding sentence.
E. Death Penalty  only permissible for homicides
1. Only permissible if imposed under a statutory scheme that gives the judge or jury reasonable
discretion, full information concerning defendants, and guidance in making the decisions.
a. Discretion: jury must be allowed to consider mitigating circumstances  e.g. the state
cannot make the death penalty mandatory if certain criteria is met (i.e. for felony murder).
b. Statute providing for the death penalty may not be vague
2. Mitigating factors
a. D is allowed to present anything he wants, basically.
3. Aggravating factors
a. D’s prior crimes
4. Victim impact statement OK
5. It is unconstitutional to impose the death penalty on the following groups
a. Minors (Roper)
b. Insane
c. Incompetent/Mentally retarded

VIII. APPEAL
A. Right to Appeal  apparently there isn’t one in the Constitution.
B. Right to Counsel
1. First appeal: yes
2. Discretionary two-tier system: only on first appeal

IX. COLLATERAL ATTACKS ON CONVICTION


A. Availability
1. After appeal is no longer available or is unsuccessful
2. D can attack conviction by beginning new, separate civil proceeding  application for writ of
habeas corpus.
B. Habeas Proceedings
1. No RTC
2. Burden of proof on D by a preponderance
3. State may appeal
4. D must be in custody to get to do this, but being out on bail or on probation or parole is
sufficient.

X. DOUBLE JEOPARDY
A. Availability
1. DJ Attaches:
a. Jury trial: when jury is empaneled and sworn
b. Bench trial: when first W is sworn
2. Exceptions that will permit a retrial:
a. Hung jury
b. Mistrial for manifest necessity
c. Mistrial because of D’s motion on any grounds not constituting an acquittal on the merits
d. D’s successful appeal
i. State may retry D unless the reversal was granted because there is insufficient
evidence to support a guilty verdict. D cannot be retried for a more serious offense
than the one he was convicted of, but a harsher sentence on retrial is permitted.
B. Same offense
1. Test used: Blockburger test  two crimes do not constitute the same offense if each crime
requires proof of an element that the other does not require proof of. If this is the case, double
jeopardy does not attach.
2. Separate punishments for same crime
a. Permissible if it is the clear intent of the legislature that the offenses carry distinct
punishment (i.e. D sentenced separately for robbing bank and for violating a “use a gun, go
to jail” statute).
C. Retrial for Different Offenses
1. Retrial for lesser offense after DJ attaches to greater one is barred.
2. Retrial for greater offense after DJ attaches to lesser one is barred, EXCEPT
a. If D is retried for murder after agg. batt, etc. V dies; or
b. New evidence surfaces of the unlawful conduct that is subsequently used to prove the
greater offense that was not known to state at the time of prosecution or has not been
discovered despite due diligence.
3. Prosecution for conspiracy okay even if some of the overt acts have already been prosecuted
4. Using evidence of a substantive offense as prior act evidence does not bar trial for that offense.
5. Can use it as a sentence enhancer.
C. Separate Sovereigns
1. The constitutional prohibition of double jeopardy does not apply to trials by a separate sovereign
 D may be tried on the same conduct by both a state and federal court, or by two states. But not
by a state or its municipalities.

X. PRIVILEGE AGAINST COMPELLED SELF-INCRIMINATION


A. Fifth Amendment Privilege
1. The government may not compel anyone to give self-incriminating testimony
2. Applies only to natural persons (not corporations), and only to statements (not an SDT for
records, etc).
B. Method for Invoking
1. D: has a right to invoke it out of the jury’s presence
2. W: will invoke it on the stand as to specific questions
C. Elimination of the privilege
1. State can compel you to testify if they grant you immunity
2. Types of immunity
a. Transactional (absolute) immunity: granted immunity for the whole transaction
b. Use & derivative use immunity: prosecutor promises that after you testify, he will never
use the testimony against you, nor any evidence that derives from your testimony  can
still prosecute based on independent evidence.
3. SCOTUS has held that a grant of use and derivative use immunity is sufficient to establish the
privilege.
4. Grant of immunity from one sovereign will preclude any sovereign from using that
testimony/evidence.

LOUISIANA STATUTORY CRIMINAL PROCEDURE

I. JURISDICTION
A. Geographic
1. Venue/jurisdiction (these are the same thing) is in the parish where the offense was committed.
2. If any act constituting the offense occurred in more than one place in or out of the parish or state,
the offense is deemed committed in any parish where the act or element occurred.
3. Special rules:
a. Murder: if the place of the crime is uncertain, where the body is found.
b. Vehicles: any parish where the offense possibly could have been committed.
c. Water: middle of the body of water is the boundary
d. Boundary/parish line: if within 100 feet, venue is proper.
5. Raising improper venue
a. Done by a pretrial motion to quash
b. If objection is not raised, it is waived.
c. Burden is on state by preponderance.
B. Change of Venue
1. Either state or D may move for change of venue based on proof that because of prejudice or
undue influence, a fair and impartial trial cannot be obtained.
2. Factors considered:
a. Nature and degree of pretrial publicity.
b. Connection of govt. officials w/ the release of publicity
c. Length of time between publicity and trial
d. Severity and notoriety of offense
e. Area from which jury is drawn
f. Other events that affect/reflect the attitude of the community or individual jurors toward D
g. Any factor likely to affect candor and veracity of prospective jurors on voir dire
h. Whether the publicity was factual or inflammatory.
3. Will be transferred to a parish where the parties can get a fair trial
4. Trial run voir dire is permissible.
C. Jurisdiction of Courts
1. Original jurisdiction
a. District Courts: state felonies and misdemeanors
b. Parish and City Courts: offenses not punishable at hard labor, parochial and municipal
ordinances within their respective territories.
i. Multiple misdemeanors joined in same indictment/information: maximum penalty
is six months of $1,000  so no right to jury trial.
c. Mayor’s courts: ordinance violations
d. Justice of the Peace courts: littering
2. Appellate jurisdiction
a. Louisiana Supreme Court:
i. Death penalty imposed
ii. Statute declared unconstitutional
b. Courts of Appeal: appeals from district courts in all other cases triable by jury
c. District Court: appeals from parish, mayor, and city court.
3. Loss of Trial Court jurisdiction: once order of appeal is entered, except to
a. Correct illegal sentence
b. Impose sentence under habitual criminal statute.

II. INHERENT POWERS OF THE COURT


A. Contempt
1. Louisiana courts can hold people in contempt for conduct that obstructs or interferes with orderly
administration of justice, impair dignity of court or respect for its authority.
2. Types of contempt:
a. Direct: conduct in presence of judge  no need for separate punishment hearing
b. Constructive: conduct not observed by court that requires proof  must give notice and
hearing

III. PRETRIAL PROCEDURES


A. Arrest
1. Defined: arrest is the taking of a person into custody by another. Either:
a. Actual stop: physical force or submission
b. Actual imminent stop*: police attempt to seize D but D neither submits nor is physically
contacted by the police  focus is on degree of certainty that D will be stopped. Factors:
i. Proximity
ii. Surrounded
iii. Weapons drawn
iv. Whether D or police on foot or in vehicle
v. Location and characteristics of area
vi. Number of police officers
*LA recognizes 4th Amendment protection a bit earlier than SCOTUS does.
2. Warrant
a. Issued by magistrate (judge with jurisdiction, justice of the peace)
b. Affidavit: used to determine PC
c. Execution: only police with territorial authority (if in pursuit, may cross parish lines and
make the arrest)
3. Private persons: may make an arrest only when D has committed a felony
4. Warrant requirement
a. Can only arrest w/out warrant if you have PC to believe that they committed a crime,
unless you are entering a residence to make the arrest.
5. Reasonable use of force is okay when making a lawful arrest.
6. Stop & frisk: LA gives statutory authority to stop and frisk (same as constitutional standard)
7. Must give Miranda warnings when D has been arrested or detained; subject to a public safety
exception.
B. Booking
1. D must be booked after arrest.
2. Must tell D:
a. Charge
b. Right to communicate w/ and procure counsel
c. Right to a preliminary examination when charged w/ a felony
3. Must also give a book information summary to the person accepting custody: name, charge, short
statement of facts, other persons arrested, or reasons why the information cannot be supplied.
C. Extradition
1. Procured by getting the governor to request the governor of the state where the person is located
to request that he be turned over to Louisian officers.
2. State must submit documents showing that the person is wanted in LA
3. D is entited to a hearing to show that he is not the person sought
D. Right to Counsel
1. LA gives right to counsel anytime a sentence of imprisonment could be imposed.
2. D may waive RTC and proceed pro se if the court determines that he is competent to do so.
3. LA Const. says that D has RTC “at each stage of the proceedings”  attaches no later than first
court appearance or judicial hearing even though D was not yet indicted
4. From moment of arrest, D has right to procure and consult w/ counsel and use a telephone to
communicate w/ friends or counsel.
5. D in custody must be brought in front of judge for appointment of counsel w/in 72 hours of
arrest.
i. Effect of noncompliance is release
ii. Appointment of counsel even if after 72 hours will cure the defect.
6. At Grand Jury
i. W: no RTC
ii. “Target W”: has RTC
a. Counsel cannot object, or address or argue before the grand jury
b. If evidence is obtained from a witness who later becomes a target, it cannot be
used against him, but W can waive this right.
E. Confessions
1. LA follows corpus delicti rule: must be independent proof of crime besides D’s confession
F. Motion to Suppress Evidence
1. General
a. D must file pretrial motion to suppress inadmissible evidence
2. Standing
a. Any person adversely affected by search and seizure has standing to raise its illegality in
the appropriate court  not just D.
b. Standing isn’t unlimited: evidence seized illegally from X used together w/ other
evidence to support SW for Y is not suppressible.
3. Procedure
a. Must file motion w/in 15 days after arraignment, unless facts are unknown or D is
otherwise excused
b. Burden of proof
i. Warrantless search: state has burden of showing PC and exception to SW
ii. Confession: state has burden beyond reasonable doubt
iii. All other cases: D has burden
c. Suppression hearing held
i. D can testify w/out being cross examined on other matters (different from broad
cross at trial)
ii. D’s testimony may not be used by the state in their case-in-chief but may be used
to impeach.
d. Failure to file motion  waiver
i. Adverse ruling carries over to trial.
ii. For confessions, state must introduce circumstances under which it was given. D
can do this also  goes to weight/credibility.
G. Identification Evidence  in CCP outline
H. Preliminary Examination
1. D arrested for a felony has a right to a preliminary examination unless or until he has been
indicted by a grand jury.
2. Purpose: to determine if there is PC for the charge in the bill of information
3. Form: both sides can call Ws, who will be subject to cross examination. D may not call child-
victim as a witness without permission of the court.
4. Can be held after indictment to preserve testimony (but D has no right to one after indictment)
5. Effect of finding no PC: fix bail
6. Warrantless arrest:
a. SCOTUS rule: 48-hour Gerstein hearing
b. Louisiana rules:
i. After warrantless arrest, officer must promptly complete PC affidavit and submit it
to the magistrate.
ii. Person in custody is entitled to a PC determination after arrest.
iii. Determination made by magistrate  no adversary proceeding. D’s presence not
required.
iv. Determination made on affidavit or other written evidence
v. This determination does not affect D’s right to preliminary examination.
vi. If PC determination is not made as provided, D released on own recognizance.
I. Bail
1. Right to bail
a. LA Const. and Code of Crim. Pro. provide for right to bail.
b. Prior to conviction:
i. Capital cases: bail is neither required nor prohibited
ii. Non-capital cases: D has a general right to bail but it can be denied for some
violent or drug crimes if there is a lot of evidence against D  state must prove by
C&C evidence that D is a flight risk or a threat to a person or the community.
c. After conviction: bail allowed if sentence is less than 5 years (or offense is punishable by
sentence of less than 5 years). Not allowed if court finds dangerousness
2. Bail is set individually in felony cases. Bail schedules are allowed in misdemeanor cases and
noncapital felonies, except in Orleans Parish.
3. Methods of satisfying bail:
a. Surety bond
i. Judges, attorneys, and ministerial officers cannot be sureties
b. Personal surety
c. Cash bond
d. Personal bond undertaking (similar to ROR)  not allowed for crimes of violence
e. Anything else deemed appropriate by the courts.
J. Instituting Prosecution
1. Methods
a. GJ Indictment: required for
i. First- and second-degree murder
ii. Agg. rape
iii. Agg. kidnapping
b. Information: other prosecutions in district court
c. Affidavit
2. Charging instruments
a. Indictment
i. Written accusation of crime made by grand jury
ii. Requires concurrence of 9 grand jurors
iii. Must be endorsed “true bill”
iv. Must be signed by foreperson
b. Information: written accusation of crime made by DA or city prosecutor
c. Affidavit: written accusation of crime made under oath and signed by affiant
3. If GJ does not indict, DA may re-indict or charge by information (if allowed for the type of
crime).
J. Grand and Petit Jurors
1. Qualifications
a. 18 years old
b. Read, write, speak English
c. Not be an interdict or incapable because of mental/physical infirmity
d. US and LA citizen in parish for a year
e. Not under indictment for felony or convicted of felony for which not pardoned
2. Exempt: people who have recently served on juries
3. Venire challenge: same as in CCP outline: exclusion of significant group
K. Grand Jury Proceedings
1. Number
a. 12 plus 2 alternates
b. 9 for quorum and to indict
2. Proceedings are secret
3. Transcripts generally not admissible at trial
4. Only people who can be present are:
a. DA or ADA
b. AG or AAG
c. Witness
d. Court reporter
e. Counsel for target W
5. Grand Jurors may not divulge testimony or deliberations except for:
a. Irregularities
b. Perjury
6. W may discuss testimony with D atty or DA.
7. GJ transcript is not admissible at trial.
L. Sufficiency of indictment or information
1. Required contents:
a. Notify D what he is charged with
b. Enable D to enter a plea to the charge
c. Define the parameters of relevancy of the evidence at trial
d. Protect D against double jeopardy
2. Special rule for gun crimes
a. When DA plans to invoke a sentencing enhancement (firearm used during crime of
violence), he must give D notice in the indictment as a prerequisite to invoking this seciond
M. Bill of Particulars
1. Method for D to seek information missing from an indictment that fails to contain sufficient
facts.
2. Done by filing a pretrial motion for a Bill of Particulars seeking who, what, where, when
information.
3. Not a discovery device  just basic information
N. Amending an Indictment
1. Technical errors  can amend anytime
a. Date
2. Error of substance  can amend prior to trial. D may move for continuance. If error discovered
after trial commences, judge shall declare a mistrial.
a. Essential allegation
3. If D fails to raise the issue or court rules against him, D’s conviction will not be set aside unless
he shows prejudice.
O. Joinder in Same Indictment
1. Of offenses
a. Same or similar character
b. Same act or transaction
c. Must be triable by same mode of trial (6 jurors, 12 jurors, etc)
2. Of Ds
a. Alleged to have participated in the same act or transaction or series of transactions
b. Not all Ds must be charged in all counts
3. Misjoinder: joinder when it is not proper as discussed above  dealt with by filing motion to
quash
4. Severance: when offenses are properly joined but as a practical matter they should be tried
separately  dealt with by filing a motion to sever
a. Court will grant the motion for severance if the joinder is prejudicial:
i. Likelihood of jury confusion
ii. D inability to make defense (i.e. can’t testify about one crime without seriously
incriminating himself on another count).
iii. Ds have antagonistic defenses (i.e. Bruton issue, one D will blame the other)
5. Additionally, D may move to consolidate two or more indictments.
P. Pretrial Attacks of Indictment  motion to quash
1. Must be made w/in 15 days after arraignment
2. Motion can NOT be used to test the sufficiency of the evidence
3. Grounds for motion to quash:
a. Invalidity of statute which creates the offense charged
b. Failure to comply w/ rules governing indictment forms
c. Indictment is duplicitous
d. Misjoinder
e. Failure of DA to furnish bill or particulars when ordered to do so by court
f. Bill of particulars shows ground for quashing
g. Double jeopardy
h. Time for instituting prosecuting has expired
i. Time for commencing trial has expired
j. Court has no jurisdiction
k. Jury was defective/improperly drawn
l. Special GJ indictment grounds:
i. Manner of selecting GJ venire or GJ itself was illegal
ii. Individual grand juror not qualified
iii. Less than 9 jurors concurred in indictment
iv. Indictment not endorsed a “true bill” or not signed by foreperson
m. Special information grounds
i. Not signed by (A)DA
ii. Inappropriate for offense charged
Q. Arraignment
1. Where D shows up and the indictment/information is read to him and he responds by pleading
2. Pleas
a. Guilty
b. Not guilty
c. NGBRI
d. Nolo contender  may only be made at court’s discretion
3. Entering pleas
a. Felony: must be in person
b. Misdemeaors: can plead through counsel
4. Guilty plea:
a. Ineffective when not knowingly and voluntarily made
b. Effect is that D states he did the crime and admits guilt of the substantive crime
5. If D pleads guilty, he must be Boykin-ized
a. Court personally addresses D on the record to make sure the plea is knowingly and
voluntarily and intelligently made. Must advise D of:
i. Privilege against self-incrimination
ii. Right to trial (and right to jury trial if applicable)
iii. Right to confront accusers
b. Court must also ascertain that D:
i. Knows his rights and waives them
ii. Understands
a. Charge
b. Consequenes of pleading guilty
c. Sentence authorized by statute (does not apply to collateral consequences,
i.e. possible deportation)
6. Special rules
a. Can’t plead guilty to a felony w/in 48 hours of arrest
b. Can’t plead guilty in a capital offense unless DA takes death penalty off the table.
c. Can’t reference a withdrawn plea at trial
R. Time Limitations
1. Sources:
a. US Const. right to speedy trial
i. Deals w/ period from commencement of proceedings to trial
ii. Undue delay between investigation and prosecution may violate due process, but
only if actual prejudice is shown and the reasons were so unjustifiable as to deviate
from fundamental conceptions of justice.
b. LA Const. right to speedy trial
c. Time limit for instituting prosecution
i. Deals w/ time period from crime to charge. Per LA Code of Criminal Procedure:
Type of Crime Time Limit
Capital punishment N/A
Life imprisonment N/A
Forcible rape N/A
Felony necessarily punishable by imprisonment at hard labor 6 years
Felony not necessarily punishable by imprisonment at hard labor 4 years
Sexual offense against V under 17 30 years from V’s 18th birthday
Misdemeanor punishable by fine or imprisonment 2 years
Misdemeanor punishable only by fine 6 months
ii. Interruption of time limitation (period begins to run anew when):
a. D, for purpose of avoiding apprehension, flees the state, is outside the
state, or is absent from usual place of abode w/in state
b. D found mentally incompetent to proceed
c. Charge dismissed by DA  will have the longer of 6 months or time
remaining.
d. Time limits for commencement of trial
i. Deals with time between charge and trial
ii. Per LA Code of criminal procedure:
Type of Crime Time Limit
Capital 3 years from institution of prosecution
Non-capital felony 2 years
Misdemeanor 1 year
iii. Interruption of Time Limitation:
a. Similar rules as above
b. D fails to appear at any proceeding after being given notice
iv. Suspension of time limitation:
a. D files motion to quash, change of venue, or other preliminary plea  time
will be suspended until the court rules. State will then have the greater of the
time remaining or one year.
e. LA Speedy Trial Act
i. D arrested  state must bring indictment/information w/in:
D in custody D not in custody
Felony 60 days 150 days

120 days for crime


punishable by life/death
Misdemeanor 45 days 90 days

ii. After the indictment or information has been brought, D must be arraigned within
30 days.
iii. Once D files motion for speedy trial, state must commence trial w/in ______
days:
D in custody D not in custody
Felony 120 days 180 days
Misdemeanor 30 days 60 days
iv. Remedy: discharge from custody unless just cause is shown
a. D who has been released may be rearrested if proceedings are
subsequently commenced.
b. If D acquiesces in delay, there is a credit day-to-day in the time limits.
S. Double Jeopardy  see CCP outline
T. Mental Incapacity to Proceed
1. Present lack of capacity
a. Prosecution may not proceed if D presently lacks the capacity to understand the
proceedings against him or assist in his defense as a result of mental disease or defect.
b. Court or either party can raise this  when raised, all proceedings grind to a halt.
c. Court will then appoint a sanity commission who will evaluate D and issue a report,
which will discuss:
i. D’s ability to understand proceedings
ii. D’s ability to assist in own defense
iii. D’s need for hospitalization
d. Lack of memory does not constitute incapacity
e. Issue of competence is determined at contradictory HG
f. Burden of proof to prove incompetence is on D.
g. If D is found competent, the prosecution resumes. If not, he is committed.
2. Exculpatory insanity
a. D does not know difference between right and wrong at the time of the crime
b. Similar procedure: sanity commission used
c. Burden on D by a preponderance
d. Court may adjudicate D guilty by reason of insanity if DA consents and court makes a
finding that there is a factual basis for the plea.
e. In capital cases, D has right to his/her own expert D witness.
f. If D is found guilty by reason of insanity:
i. Capital case: D committed to state or private mental institution.
ii. Court holds commitment proceedings: Is D danger to self or others?
U. Recusal of Judge
1. Mandatory grounds:
a. Judge is biased, prejudiced, or personally interested in a case to the extent the he would be
unable to conduct a fair trial
b. Judge is “related closely” to D, V, D atty, or DA
c. D was involved in the case as an attorney or was associated with an attorney involved in
the case
d. Judge is material W
e. Judge has performed a judicial act in the case in another court
f. Judge would, for any other reason, be able to conduct a fair and impartial trial.
2. Judge may recuse self on own motion or either party can file one.
V. Recusal of DA
1. DA shall be recused when:
a. Has personal interest in case
b. Is related to D or V
c. D has been employed or consulted in the case as attorney for D
2. Raised by filing a motion to disqualify
W. Dismissal of Prosecution
1. DA has unfettered discretion to dismiss indictments.
2. Effects of dismissal
a. Will discharge the indictment or count
b. Not a bar to subsequent prosecution unless double jeopardy has attached.
X. Continuance
1. Motion for continuance must be:
a. In writing
b. Specifically state the reasons why the continuance is sought.
c. Filed at least 7 days before trial (court can waive this for good cause)
2. Continuance may affect right to speedy trial
3. If based on absence of W:
a. State that W was expected to testify
b. Materiality of testimony and necessity for W’s presence at trial
c. Probability that W will testify and be available at next trial date
d. That due diligence was used to procure W’s presence
4. It is rare that court will be considered to have abused its discretion in denying a continuance 
even if state and D Atty agree, court can still deny.
Y. Discovery
1. D must file motion for discovery to get this stuff.
2. What D is entitled to:
a. D’s statements
b. D’s criminal record
c. Documents and tangible objects
d. Reports of examinations or tests
e. Evidence of other crimes
f. Hearsay statements of co-conspirators
g. Confessions and inculpatory statements of co-Ds
h. Initial police report
i. Exculpatory evidence
3. D is not entitled to:
a. DA/Police work product
b. W statements
c. Names/addresses of Ws
4. Discovery by state  state must file motion for discovery
a. Mental disease/defect  must give notice of intent to use w/in 10 days of trial
b. Alibi  D must notify within 10 days of motion for discovery
i. Must contain time, place, and names and addresses of Ws
ii. Failure to comply may result in exclusion of W’s testimony
c. Reports of examinations or tests
d. Documents or tangible objects
5. Sanctions: anything within trial court’s discretion except for dismissal (usually continuance or
exclusion).

IV. TRIAL
A. Opening statements
1. Can be waived
2. State must notify D of intent to use confession or inculpatory statement in opening statement
3. Evidence not within scope of opening statement is inadmissible
B. Misc.
1. Jurors can take notes only if DA and D Atty agree in open court
2. Cases are randomly allotted to judges  violates due process if DA is permitted to assign cases
3. Sur-rebuttal is permitted at judge’s discretion in the interests of justice
4. Right to public trial
C. Mistrial
1. Grounds:
a. Comment made within hearing of jury by judge, DA, or court official during trial or
argument that refers directly or indirectly:
i. D’s race, religion, color, national origin if common not material or relevant and
might create prejudice
ii. Another crime committed by D which is not admissible
iii. Failure of D to testify on his own behalf
iv. Refusal of judge to direct a verdict
b. When D consents
c. Hung jury
d. Legal defect which would make verdict reversible as a matter of law
e. Lack of mental capacity by D to proceed
f. Physically impossible to proceed.
g. False statement of a juror on voir dire prevents a fair trial
h. Upon motion of D  prejudicial conduct makes it impossible for D to obtain a fair trial
i. State and D jointly move for mistrial
D. Motion for Judgment of Acquittal
1. Only allowed during bench trials, D may move after close of state’s evidence
E. Jury vs Non-Jury Trial
1. Capital cases: 12 person jury/unanimity
2. Cases Necessarily Punishable at HL (Felony): 12 jurors/10 for verdict
3. Relative Felony: 6 jurors/unanimity
4. Misdemeanor where punishment may be a fine in excess of $1,000 or imprisonment for more
than 6 months: 6 person jury/unanimity
5. Other misdemeanors: tried by judge
6. Joinder of misdemeanors: the potential cumulative penalty determines
F. Waiver of Jury Trial
1. D may do so knowingly and intelligently
2. Court must advise D of this right
3. Can withdraw waiver unless doing so would interfere w/ administration of justice, delay
proceedings, inconvenience witnesses, etc.
4. Must be waived w/in 15 days of arraignment
G. Jurors
1. Sequestration:
a. Capital: sequestered the whole time
b. Non-capital: after jury is charged, whenever the court orders
2. Challenges
a. For cause: must be made before indictment is read to jury
i. Juror lacks qualifications
ii. Juror is not impartial
iii. Related to D, V, DA, or D Atty to the point that it would influence them
iv. Will not accept law as given to them by court
v. D served on grand jury or prior petit jury that heard the same same
vi. Juror biased against statute
vii. Won’t vote for death penalty in penalty case
viii. Will not convict on circumstantial evidence
b. Peremptory: must be made before jury is sworn; backstrikes allowed
3. Number of peremptories
a. Capital and felonies necessarily punishable by imprisonment at hard labor: 12 each for
state and all Ds
b. Other: 6 each
H. Right to Present a Defense
1. Compulsory process: D is entitled to subpoena witnesses who will testify on his behalf at the
state’s expense.
2. Confrontation of Ws
3. Privilege against self-incrimination
4. Presumption of innocence until proven guilty by reasonable doubt
I. Sentencing
1. LA sentencing guidelines exist: not mandatory, but by statute, judge must say he considered
them.

V. POST-CONVICTION PROCEDURES
A. Motion for New Trial
1. Court should grant if:
a. Verdict contrary to law and evidence
b. Court’s ruling on a written motion or on objection made during the proceedings shows
prejudicial error.
c. Newly discovered evidence has become available which:
i. Must be material
ii. Could not have been discovered previously by reasonable diligence; and
iii. Its introduction probably would have changed the verdict
d. Discovered of prejudicial error not previously discovered by reasonable diligence
e. Ends of justice demand it  at judge’s discretion
B. Motion for Post-Verdict Judgment of Acquittal
1. D may move for this before sentencing (sentence may not be imposed until at least 3 days after
conviction)
2. Standard: trial court must view the evidence in the light most favorable to the state, and grant the
motion if it finds that the evidence does not reasonably permit a finding of guilty.
3. Standard for reviewing: appellate court must set aside the verdict if no reasonable jury could
have found guilt beyond a reasonable doubt
4. If trial court finds that the evidence only supports a lesser included or responsive offense, it
should modify the verdict to reflect that
5. State may obtain appellate review of the granting of this motion:
a. Appellate court may reinstate it
b. No double jeopardy problem because no need for retrial
c. D may not be retried if appellate court sustains the judgment or sets aside the verdict
C. Motion in Arrest of Judgment
1. D may move for this before sentencing.
2. Court shall arrest judgment only when:
a. Indictment is substantially defective in that an essential averment is omitted (requires
showing of prejudice)
b. Offense charged is not punishable under a valid statute
c. Court is w/out subject matter or geographnic jurisdiction (if not waived)
d. Tribunal that tried case did not conform to the requirements requiring the appropriate
mode of trial (jury, non-jury, number of jurors, etc).
e. Verdict is not responsive to the indictment or is otherwise defective
f. Double jeopardy
g. Prosecution not timely instituted
h. Capital or life sentence not instituted by GJ indictment
3. Effect: can’t commence new proceedings unless legally permissible under the circumstances
D. Motion to Reconsider Sentence
1. D may file within
a. Felony: 30 days of sentencing
b. Misdemeanors: anytime following commencement or execution of sentence
2. Permissible form
a. Oral if at time of sentencing
b. Otherwise in writing
3. Trial court may resentence even if appeal is pending
4. Failure to file the motion and include a specific ground will bar that ground from being
challenged or raised later
E. Appeals
1. Timing: D has 30 days from rendition of judgment or ruling on motion to reconsider sentence in
which to make an oral or written motion for an appeal
2. Can only appeal a final judgment or ruling
3. State can appeal ruling on:
a. Motion to quash
b. Time limitations
c. Double jeopardy
d. Arrest of judgment
e. Change of venue
f. Recusal
4. D may appeal
a. Judgment which imposes a sentence in a case triable to a jury
b. A ruling on a motion by the state declaring D’s present sanity
5. Standard of Review: if all evidence (admissible and non-admissible) is insufficient to support
conviction, D may be discharged. When all evidence (admissible and non-admissible) is insufficient
but court finds prejudicial error, conviction is set aside and new trial is permitted even if the
admissible evidence alone is insufficient.
6. Rehearing: party has 14 days from the disposition of appeal. If no application is filed or the
application is denied w/in 14 days, the judgment becomes final.
F. Post-Conviction Relief
1. Statutory provisions in La. Code of Criminal Procedure.
2. Only available when other forms of relief are not available AND D is in custody (substitute for
habeas corpus.
3. Grounds:
a. Conviction obtained in violation of US or La. Constitution.
b. Court exceeded its jurisdiction.
c. Double Jeopardy
d. Limitation on institution of prosecution has expired
e. Statute creating the offense is unconstitutional
f. Conviction or sentence violates prohibition against ex post facto laws
4. Must be filed within 2 years of judgment of conviction becoming final, unless
a. Facts not known to D or D atty
b. Based on appellate decision involving retroactive application of constitutional rule
c. D was sentenced to death
5. State may raise procedural objections
6. D entitled to evidentiary hearing only when factual questions cannot be resolved by summary
procedures
7. When indigent Ds have a colorable claim for relief, they will be entitled to counsel
G. Habeas Corpus
1. Limited to pre-conviction, unlawful arrest, detention in mental institution, etc.
2. Grounds – court ordered custody:
a. Court exceeded its jurisdiction
b. Although original custody was lawful it has since become unlawful
c. Order of custody legally deficient
d. Improper custodian
e. Custody prior to trial violates due process where there is no court order if person is being
held in custody unlawfully

EVIDENCE

I. WITNESSES
A. Competency
1. Requirements to testify
a. W cannot testify to a matter unless he has personal knowledge of it
b. W not testifying as an expert may only testify in the form of an opinion or inference only
where they are rationally based on W’s perception and helpful to a clear understanding of
his testimony or the determination of a fact at issue
2. Jurors cannot be witnesses, but may testify about improper outside influence and extraneous
prejudicial information (criminal cases)
3. Judges can also not be witnesses
B. Experts
1. Form of testimony
a. W who is qualified as an expert by knowledge, skill experience, training, or education
may testify in the form of an opinion or otherwise if scientific, technical, or other
specialized knowledge will assist the trier of fact to understand the evidence or determine a
fact at issue.
b. Experts may not testify to D’s guilt or that he acted with “specific intent to kill.”
c. In criminal cases, an expert must state the facts on which his opinion is based. In civil
cases, expert W does not.
i. If these facts are otherwise inadmissible, they may only be developed on cross-
examination.
d. Expert may not testify as to the credibility to a witness/V but may be used to rebut an
attack on the witness’s testimony.
2. Threshold test: whether the methods, data, etc. used by the expert has sufficient indicia of
reliability. Daubert standard  judge is a gatekeeper and keeps out any evidence that does not
meet the criteria. Factors to consider:
a. Testability of expert’s theory and technique
b. Whether the theory or technique has been subject to peer review and publication
c. Known or potential rate of error
d. General acceptance
C. Credibility
1. Cannot corroborate or impeach until W sworn.
2. Can impeach own W.
3. Attacking credibility
a. Prior convictions
i. Criminal: any crime, any time
ii. Civil: crimes punishable by death or imprisonment for more than 6 months within
ten years
iii. Pardon based on innocence will preclude the use of a conviction. Appeal does
not.
iv. Adjudication of juvenile delinquency is generally inadmissible, unless D’s due
process rights to a fair trial require its admissible
v. When Details can be used:
a. When W has denied conviction or recollection thereof
b. When W has testified to exculpatory facts thereto
c. When probative value outweighs danger of unfair prejudice, confusion of
the issues, or misleading the jury
b. Bias, interest, corruption
i. Can only show bias/interest/corruption against or for a party.
c. Prior contradictory statement
i. Laying a foundation
a. Ask W if he made statement
b. If W does not distinctly admit statement, ask if they made the prior
statement
ii. This is used to attack credibility, not for proof of the facts stated
iii. When W’s testimony is attacked as to particular fact, prior consistent statements
are admissible
d. Bad reputation for truth and veracity
i. Call character W
ii. Lay foundation: ask W if he heard discussion of reputation for truth and veracity
in a relevant or discrete community
iii. Can’t ask about particular acts or W’s own opinion
4. Cross examination
a. LA has wide-open cross
b. D who takes stand waives privilege against self-incrimination and is subject to full cross
c. D cannot restrict scope of cross by only testifying to facts in confession
5. Court can’t call its own witnesses on their own motion unless
a. Authorized by legislation; or
b. Requested by a party
D. Exclusion of witnesses
1. Judge may sequester witnesses on its own motion but must do so if requested by a party.
2. Exceptions:
a. Party
b. Single officer or employee of a party who is not a natural person designated by counsel as
its representative or case agent
c. Person whose presence is shown by a party to be essential to the presentation of the case
(i.e. expert)
d. V of crime or V’s family member

III. RELEVANCY
A. General
1. Relevant evidence: any evidence having a tendency to make the existence of any fact of
consequence to the determination of the action more probable than it would be without the
evidence.
2. RULE: Only relevant is admissible.
B. Limitations
1. Danger of unfair prejudice
2. Confusion of the issues
3. Misleading the jury
4. Considerations of undue delay or waste of time
5. In civil cases: offers of compromise, payment of medical or other expenses
C. Relevancy of Character Evidence
1. RULE: Evidence of a person’s character is not admissible to prove that he or she acted in
conformity therewith on a particular occasion, subject to two exceptions:
a. Criminal D’s character: based on reputation in community
i. D may show good character of pertinent trait (i.e. honesty in theft case)
ii. State may rebut only and show bad character
iii. State may cross-examine defense Ws as to whether they heard about specific bad
acts.
b. V’s character
i. D may not introduct evidence of C’s dangerous characters or threats against D
unless D shows hostile demonstration or overt act by V at time of offense charged
ii. Exception: not required in DV situation  to protect victims of domestic violence
iii. Only general reputation evidence allowed
D. Past Misconduct
1. Generally not admissible to prove D’s guilt or propensity.
2. However, is admissible to prove
a. Knowledge
b. Intent
c. Plan
d. Purpose
e. Opportunity
f. Motive
g. Identity (i.e. unique/signature crime, system)
h. Absence of mistake
i. Res gestae (integral part of same act that is part of the act or transaction that is the subject
of the present proceeding)
3. Independent Relevance
a. 412.2 Evidence of Prior Sex Crimes
i. Lustful disposition evidence for specific intent sex crimes
ii. Exception: rape charge
4. Homicide D offering V’s alleged past misconduct
a. Generally, he cannot
b. Can do so if there was a hostile demonstration or overt act at the time of the offense
charged
5. Procedure for using prior bad acts evidence:
a. Provide reasonable pretrial notice
b. Specify the basis for admissibility
c. Show that the evidence isn’t cumulative
d. The evidence truly serves the purpose for which it is offered
e. Establish by same standard required in FRE (preponderance?)
f. D entitled to limited jury instruction
E. Misc. Evidence and its Admissibility
1. Habit/routine: relevant to prove that the person or organization acted on a particular occasion in
conformity w/ the habit or routine practice
2. Guilty pleas that are later withdrawn: inadmissible
3. Nolo pleas: inadmissible
4. Insurance police: may be admissible, but the amount of coverage shall not be told to the jury
unless it’s a disputed issue
5. Blanket rule against aadmissibility of lie detector tests
6. Admissible in criminal cases but not in civil cases:
a. Subsequent remedial measures
b. Compromise or offer to compromise a claim on the issue of the validity of the claim
c. Furnishing or offering to pay expenses or losses resulting from injury to person or damage
to property on the issue of liability for injury or damage
d. An amount paid in settlement or tendered for settlement except on the issue of failure to
make settlement
e. Nature and extent of worker’s claim or payment of past or future worker’s comp claims

IV. HEARSAY
A. Definition
1. Hearsay: an out-of-court statement offered to prove the truth of the matter asserted
B. Hearsay Exclusions
1. Prior statements of W  declarant testifies at trial and is subject to cross-examination, and either
a. Statement is consistent with declarant’s testimony, and is either
i. Initial complaint of sexual assault;
ii. Offered to rebut charge of recent fabrication
b. Statement is one of identification of a person made after perceiving the person
c. Statement is offered in a criminal case and is inconsistent with testimony, provided that
i. Proponent has first fairly directed W’s attention to the statement
ii. W has been given the opportunity to admit the statement; and
iii. Additional evidence to corroborate the matter asserted in the prior statement
exists.
2. Personal, adoptive, and authorized admissions
a. Party admission
3. Relational and privity admissions
a. Employee/agent
b. Co-conspirator statement made while participating in the conspiracy and in furtherance of
the conspiracy
4. Things Said and Done (Res Gestae)
a. Statement under immediate pressure and occurrence of the criminal act (bank teller
example)
C. Hearsay Exceptions
1. Availability of Declarant Immaterial:
a. Present sense impression: statement made while perceiving event or immediately
thereafter
b. Excited utterance
c. Then existing mental, emotional, or physical condition (no memory or belief)
i. “I don’t feel well,” “I’m in pain,” etc.
d. Statement for purpose of medical treatment or diagnosis
e. Recorded recollection
i. Once new something and wrote it down  can read document into evidence
f. Records of regularly conducted business activities
i. Kept in the regular course of business activity
ii. Police and investigative reports are not admissible by govt. when govt. is a party
g. Public records and reports
h. Absence of public records
i. Records of vital statistics
j. Ancient documents
i. Old newspapers, etc
k. Learned treatises
j. Judgment of previous convictions
2. Unavailability of declarant required
a. Former testimony
i. Crawford applies
ii. Will violate right to confrontation unless:
a. D was represented by counsel,
b. W testified under oath,
c. W was cross examined or D waived right
d. W was unavailable; and
e. State made a good-faith effort to obtain presence of W.
b. Statement under belief of pending death
c. Statement against interest
i. Covers only statements that are individually inculpatory, not ones that shift the
blame (though mention of others is okay)
d. Statement of personal or family history
e. Complaint of sexually assaultive behavior
f. Statement against party who engaged or acquiesced in wrongdoing that resulted in
unavailability of the declarant as a witness  Giles
g. Civil cases: omnibus exception if court determines that the testimony is trustworthy, there
is no other evidence available, and notice is given to the other party
3. Constitutional considerations
a. D has federal/state constitutional right and LA statutory right to be confronted with
witnesses against him
b. Confrontation clause: must satisfy two requirements:
i. Reliability; and
ii. Unavailability

V. PRIVILEGES
A. Specific Relationship Privileges
1. Atty-client
a. Duration: applies during and after employment  SCOTUS says that federal privilege
continues until death of client
b. Scope: perceptions, observations, communications, mental/physical/emotional state of
client in connection with such communication.
c. Extends to both lawyer and client representatives
d. Exceptions:
i. Services of lawyer were sought to facilitate criminal activity
ii. Concerning identity of client
e. Attorney may claim the privilege on behalf of the client
i. Cannot subpoena a lawyer in a criminal case (including prosecutor) to seek
information about client or former client obtained in the course of representation
unless there is a contradictory HG and court finds:
a. Essential to investigation, prosecution, or defense;
b. Not for harassment;
c. Information sought is described with particularity; and
d. There is no other practical way of getting the information.
2. Spousal
a. Confidential communication privilege: when talking to spouse, it is off-limits to anyone
else. D can prevent spouse from talking about or answering questions about this.
i. Does not apply if one spouse is charged with crime of person or property of
another.
ii. Exists during and after marriage
b. Spousal witness privilege
i. Exists during and after marriage
ii. Belongs to W, not to D.
3. Informant: generally don’t have to disclose their identity unless it’s required by due process.
4. Police records are confidential but D can discover them and use them to impeach
B. Waiver of Privilege
1. Privilege is waived if holder of privilege voluntarily discloses or consents to its disclosure of any
significant part of the privileged matter, but not if the disclosure itself is privileged.
2. If joint holders of privilege  both need to waive

VI. AUTHENTICATION
A. Rule: must authenticate documents  show that they are what they purport to be.
B. Examples of what will authenticate a document:
1. Testimony of W with knowledge
2. Nonexpert opinion of genuineness of handwriting
3. Comparisons by tries of fact or expert witnesses with authenticated specimens
4. Distinctive characteristics
5. Voice ID
6. Telephone conversations
7. Public records or reports
C. Self-Authenticating Documents
1. Public documents
2. Official publications
3. Authentic acts
4. Commercial paper
D. Chain of custody: tangible evidence may be authenticated by showing the chain of custody
E. “Best Evidence” Rule
1. A duplicate is admissible to the same extent as an original unless:
a. A genuine issue is raised as to the authenticity of the original;
b. It would be unfair to admit the duplicate in lieu of the original under the circumstances; or
c. The original is a testament offered for probate, a contract on which the claim or defense is
based, or otherwise closely related to a controlling issue.
2. A duplicate cannot be deemed admissible simply because it is in electronic form or is a
reproduction of an electronically imaged or stored record, document, data, or other information.
3. Original is not required and other evidence of the contents will be allowed if:
a. Originals were lost or destroyed (except because of bad faith by proponent)
b. Originals cannot be obtained by any available judicial procedure or process
c. The original, when under the control of a party against whom it is offered, does not
produce the original (after being put on notice that the original would be a subject of proof)
d. The original is not closely related to a controlling issue
e. The originally cannot be practically produced in court because of its location or cost in
incurring the original is prohibitive and a copy will serve the evidentiary purposes.

You might also like